Vous êtes sur la page 1sur 54

Propriedades aritmticas dos inteiros

CAPTULO 1 PROPRIEDADES ARITMTICAS DOS INTEIROS

A Matemtica a rainha das cincias e a Teoria dos Nmeros a rainha da Matemtica.

C.F.Gauss
Objetivos do Captulo 1

(a) Descrever as propriedades da soma e do produto de inteiros; (b) Destacar a propriedade de domnio de integridade; (c) Explicitar a estrutura de ordem induzida pelo conjunto dos nmeros naturais; (d) Aplicar a teoria a problemas elementares de contagem.

1.1

INTRODUO

A Teoria dos Nmeros um vasto edifcio matemtico, repleto de resultados que ilustram a interdisciplinaridade das grandes reas de atuao da Matemtica. A parte elementar da Teoria dos Nmeros a chamada Aritmtica, que estuda as propriedades das operaes de soma e produto dos nmeros inteiros, a includo o algoritmo euclidiano da diviso e suas aplicaes. Esse desenvolvimento, iniciado pelos Os Elementos de Euclides (cerca de 300 AC) foi consolidado na Renascena, pelos trabalhos de Fermat, Euler e Gauss. Na realidade, prosseguindo na direo apontada por Gauss, os trabalhos de Kummer, Dedekind e Kronecker, favoreceram a consolidao da Teoria Algbrica dos Nmeros. Um pouco mais tarde, ainda no sculo 19, a chamada Teoria Analtica dos Nmeros incorporou mtodos de anlise real e complexa, dos trabalhos de Dirichlet e Riemann. Atualmente, uma nova vertente, a Geometria Aritmtica, fruto dos estudos de Artin, Hasse, Mordell e Weil, vem permitindo avanos notveis, notadamente, por exemplo, a demonstrao do ltimo Teorema de Fermat (Wiles, 1995). Nesse curso introdutrio, procuraremos sistematizar a parte inicial da Aritmtica, em seus aspectos mais aplicados ao bom entendimento da estrutura operacional do conjunto dos inteiros.

1.1

SOMA E PRODUTO DE INTEIROS

Representaremos por Z = {. . . , 3, 2, 1, 0, 1, 2, 3, 4, . . .} o conjunto dos nmeros inteiros e por N = {0, 1, 2, 3, . . .} o subconjunto de Z formado pelos nmeros naturais. Dados os inteiros a e b, indicaremos por a + b o inteiro que representa sua soma e por ab o inteiro que representa seu produto, ou seja: + : Z Z Z . : Z Z Z (a, b) a + b (a, b) a.b Essas operaes, tambm chamadas de caractersticas.
adio

multiplicao,

possuem as seguintes propriedades

Introduo Teoria dos Nmeros  [Antonio Carlos & Ana Paula Marques]

1) A adio e a multiplicao so bem denidas:

a, b, c, d Z, a = c e b = d = a + b = c + d e ab = cd
2) A adio e a multiplicao so comutativas:

a, b Z, a + b = b + a e ab = ba
3) A adio e a multiplicao so associativas:

a, b, c Z, (a + b) + c = a + (b + c) e (ab)c = a(bc)
4) A adio e a multiplicao possuem elementos neutros, unicamente determinados pelas condies:

a Z, a + 0 = 0 + a = a e a 1 = 1 a = a
5) Todo inteiro a Z possui um nico oposto aditivo, notado (a), tal que

a + (a) = (a) + a = 0
6) A multiplicao distributiva com relao adio:

a, b, c Z, a(b + c) = ab + ac
Essas propriedades, as leis bsicas da aritmticas, que estamos supondo conhecidas, tambm comparecem em muitas situaes, relativas a conjuntos outros que o dos inteiros. Observemos que o produto de inteiros no possuia propriedade correspondente a (5) acima, isto , no existe, em geral, para cada inteiro a, algum inteiro b tal que ab = ba = 1; na realidade, veremos que 1 1 = (1) (1) = 1 [Ver a Prop.B do 1.7]. Para contornar parcialmente essa diculdade, vale a seguinte relao.
Observaes

7) [Integridade] Dados a, b Z, se ab = 0, ento a = 0 ou b = 0. Equivalentemente, tendo em conta a formulao contrapositiva, se a = 0 e b = 0, ento ab = 0, ou notando Z = Z {0} o conjunto dos inteiros no nulos, temos, se a, b Z , ento a b Z , o que signica que Z estvel pelo produto dos inteiros.
EXEMPLO 1.3

Nesse exemplo, usaremos as propriedades (1)(7) para recordar vrios resultados. 1. Dados os inteiros a e b, a equao x + a = b possui soluo nica x = b + (a) = b a. De fato, se x + a = b = y + a, ento x + a = y + a, donde, somando (a) a ambos os membros, (x + a)+(a) = (y + a)+(a), ou, pela associatividade, x +(a +(a)) = y +(a +(a)), x +0 = y +0, x = y ; assim, s pode existir uma soluo. Enm, para exibir tal soluo, partindo de x + a = b e adicionando (a) a ambos os membros, (x + a) + (a) = b + (a), x + (a + (a)) = b + (a), isto , x + 0 = b + (a), x = b + (a). Como de hbito, notaremos b a = b + (a). Em particular, se x + a = 0, ento x = a o nico inteiro que verica tal soma. Ora, como a + (a) = 0, segue que (a) = a. 2. A adio compatvel e cancelativa com relao igualdade: a, b, c Z, a + b = a + c b = c Se a + b = a + c, ento, somando (a) a ambos os membros, temos (a) + (a + b) = (a) + (a + c), ou, pela associatividade, ((a) + a) + b = ((a) + a) + c, donde 0 + b = 0 + c, b = c. A recproca imediata da propriedade (1). 3. Para cada a Z, temos a 0 = 0. Temos a0 = a(0 + 0) = a0 + a0, isto , 0 + a0 = a0 + a0, donde, pelo cancelamento aditivo acima, a0 = 0. 4. Dados a, b Z, temos a(b) = (a)b = (ab); em particular, (a)(b) = ab.

Propriedades aritmticas dos inteiros

Como b + (b) = 0, vem a(b + (b)) = a0 = 0, vem ab + a(b) = 0, donde (ab) = a(b); analogamente, (a)b = (ab). Enm, (a)(b) = (a(b)) = ((ab)) = ab. 5. Dados a, b, c Z, temos a(b c) = ab ac. De fato: a(b c) = a(b + (c)) = ab + a(c) = ab + (ac) = ab ac. 6. [Cancelamento restrito da multiplicao] a, b, c Z, a = 0, ab = ac = b = c De ab = ac segue ab ac = 0, ou a(b c) = 0; pela propriedade da integridade (7), como a = 0, vem b c = 0, ou b = c. 7. Em Z, se a2 = 2a, ento a = 0 ou a = 2. Temos a2 2a = a(a 2) = 0, donde, usando a integridade, a = 0 ou a 2 = 0. Os cdigos para efetuar as operaes com inteiros so, certamente, conhecidas de todos ns. A aparncia algo formalizada da exposio , to-somente, o sotaque matemtico adequado para validar os mecanismos operacionais. Por exemplo, no exerccio 7. acima, lanamos mo da potncia a2 , mesmo que ainda no tenhamos (re)visto sua denio.
Observao Atividade-proposta 1.4

1. Verique a validade das relaes: (a) a = (1)a (b) (a + b) = (a) + (b) (c) (a b) + (c d) = (a + c) (b + d) (d) (a b) (c d) = (a + d) (b + c) (e) (a b)(c d) = (ac + bd) (ad + bc) (f) (a b)(a + b) = a2 b2 2. Encontre todos os inteiros x Z tais que: (a) x + 2 = 18; (b) 10x = 0; (c) 2x 3 = 0; (d) 2x + 4 = 0; (e) x2 + 1 = 0 3. Dados a, b Z, considere a equao ax = b. Comprove os seguintes resultados: (a) se b = 0 e a = 0, ento todo x inteiro soluo da equao dada; (b) se b = 0 e a = 0, ento x = 0 a nica soluo da equao considerada; (c) se a = 0 e b = 0, ento a equao no possui soluo inteira; (d) se a = 0 e se existir alguma soluo x da equao, ento tal soluo nicamente determinada; (e) mostre com um exemplo em que a condio a = 0 no suciente para garantir a existncia de soluo inteira da equao dada. 4. Encontre todos os inteiros x Z tais que: (a) x2 = 1; (b) x3 = x; (c) x2 + 2x + 1 = 0 5. Quantos inteiros x vericam 0 x 10 ? 25 x 79? 6. Indique o inteiro que ocupa a posio 53 na sequncia 86, 87, 88, . . .. 7. Dentre 123 inteiros consecutivos, o maior vale 307. Indique o menor inteiro. 8. Sejam os inteiros a e b tais que a + b = 21 e ab = 7. Calcule (a) a2 + b2 ; (b)a4 + b4 . 9. Calcule (mentalmente...) o inteiro a = 1234567892 123456790 123456788. Sugesto: x2 1 = (x 1)(x + 1);

Introduo Teoria dos Nmeros  [Antonio Carlos & Ana Paula Marques]

1.5 A estrutura de ordem

O conjunto dos inteiros possui uma ordem natural: Z = {. . . , 4, 3, 2, , 1, 0, 1, 2, 3, 4, 5, . . .}; essa ordem, habitualmente, expressa em termos da relao a < b, onde a armao a menor do que b signica que o inteiro a est esquerda do inteiro b na lista acima, isto , a diferena b a um inteiro positivo. Desse modo, as propriedades da relao a < b podem ser obtidas das propriedades do conjunto dos inteiros positivos N = {1, 2, 3, 4, . . .}. Admitiremos, assim, como postulados, as seguintes propriedades de N : (O1) Adio: a soma de dois inteiros positivos positiva; (O2) Multiplicao: o produto de dois inteiros positivos positivo; (O3) Tricotomia: para cada inteiro a, vale uma e somente uma das alternativas: ou a positivo, ou a = 0, ou a positivo. Usaremos as seguintes denies: a < b b > a b a > 0; a b signica que ou a < b ou a = b. Assim, os inteiros positivos a so os inteiros maiores do que zero; inteiros b < 0 so chamados negativos.
Propriedades

1) [Transitividade]

a, b, c Z, a < b e b < c = a < c


Como b a > 0 e c b > 0, segue de (O1) que c a = (c b) + (b a) > 0, ou seja, a < c. 2) [Adio compatvel e cancelativa] a, b, c Z, a < b a + c < b + c De fato, vale (b + c) (a + c) = b a. 3) [Multiplicao compatvel e cancelativa restrita] a, b, c Z, c > 0, a < b ac < bc Basta usar as denies e a relao bc ac = (b c)a. 4) A relao uma relao de ordem, isto , valem as propriedades: (a) Reexiva: a, a a; (b) Anti-simtrica: a, b, a b e b a = a = b (c) Transitiva: a, b, c, a b e b c = a c

Exemplo 1.6

1. Seja x Z ; vale x2 > 0. De fato, pela tricotomia, como x = 0, ento ou x ou x positivo. No primeiro caso, x2 positivo pela condio (O2); no segundo, x positivo, logo x2 = (x)2 > 0. Em particular, 1 = 12 sempre positivo. 2. Para cada a Z, o valor absoluto de a dado por: |a| = Valem as relaes: (a) |ab| = |a| |b|; (b) |a| a |a| (c) |a + b| |a| |b| Verique as trs relaes acima.

a, se a 0 a, se a < 0

Pratique um pouco!

Propriedades aritmticas dos inteiros

1.7 O princpio da Boa Ordenao

Seja S um subconjunto de N. Diremos que um inteiro a um menor elemento de S se (i) a S ; (ii) para cada x S , vale x a. Caso exista, o menor elemento de S unicamente determinado, pois se a e b so menores elementos de S , ento a b e b a, donde a = b (antisimetria da relao de ordem). Usaremos a notao minS para denotar, se existir, o menor elemento de S . Um resultado notvel garante a existncia de menor elemento de subconjuntos no vazios de N. Diremos, ento, que o conjunto N bem ordenado. Axioma da Boa Ordenao Todo subconjunto no vazio de nmeros naturais possui um menor elemento.
Proposio A

Dado a Z, se a > 0, ento a 1. Por absurdo, se existe a Z tal que 0 < a < 1, ento o conjunto S = {x N ; 0 < x < 1} no vazio. Portanto, S possui um menor elemento b tal que 0 < b < 1, donde 0 < b2 < b < 1, e seria b2 S , com b2 < b, contradizendo a minimalidade de b. Dados os inteiros a e b, se a > b, ento a b + 1. De fato, se b < a < b + 1, ento a b = c > 0 tal que b < b + c < b + 1, ou 0 < c < 1, o que impossvel.
Corolrio 2

Corolrio 1

Dados os inteiros a e b, com b = 0, vale |ab| |a|. Sejam os inteiros a e b. Se ab = 1, ento a = b = 1 ou a = b = 1.

Como b = 0, vem |b| 1, donde |a||b| |a|, ou |ab| |a|.


Proposio B

De fato, de ab = 1 vem que a = 0 e b = 0; ento, do corolrio 2 e da condio 1 > 0, vemos que 1 = |ab| |a| e 1 = |ab| |b|. Tambm, como |a| > 0 e |b| > 0, vem da propsio A que |a| = |b| = 1, ou a = 1 e b = 1; enm, da hiptese ab = 1, temos bem a = b = 1 ou a = b = 1.
Proposio C [Propriedade Arquimediana]

Dados os inteiros a e b, com b = 0, existe um inteiro

n tal que nb a.
De fato, como b = 0, segue do corolrio 2 que |b||a| = |ba| |a| a. Assim, se b > 0, basta escolher n = |a| e o resultado decorre da desigualdade anterior. Se b < 0, ento a escolha n = |a| tambm verica o resultado enunciado.

Induo Finita

11

CAPTULO 2 INDUO FINITA


Deus fez os inteiros; o resto inveno humana.
L.Kronecker

Objetivos do Captulo 2

(a) Discutir a validao de sentenas abertas em conjuntos innitos; (b) Descrever o mtodo da induo matemtica; (c) Explicitar o Teorema da Induo Finita em suas duas formas; (d) Estabecer denies por recorrncia; (e) Aplicar a teoria a problemas elementares de contagem.

2.1

INTRODUO

Como o conjunto

dos inteiros innito, ocorrem diculdades de ordem lgica e operacional para

formular e validar corretamente vrias propriedades, mesmo aquelas de carter

discreto.

A induo matemtica ou induo nita fornece critrios ecientes para estabelecer verdades matemticas, vlidas em conjuntos innitos. s vezes, uma induo emprica pode apontar para

uma certa concluso, aps vrias tentativas experimentais concordantes. Mesmo que o nmero de tentativas seja muito grande, uma induo emprica no implica, necessariamente, numa certeza. Consideremos os exemplos, onde (1) (2) (3) (4)

representa um qualquer nmero natural:

P (n) = n2 + n + 41 um nmero primo; T (n) = n(2n 1) < 6000; Se Sn a soma dos angulos internos de um n(n + 1) 1 + 2 + + n = 2 43, 47, 53, . . . , 1681,

polgono, ento

Sn = (n 2) ;

No caso (1), um exemplo de Euler, substituindo sucessivamente sequncia de nmeros primos no primo. No caso (2), procedendo por tentativas, como em (1), com de mas

P (41) =

412

n = 1, 2, 3, . . . , 40, encontramos a + 41 + 41 = 41(41 + 2) = 41 43


encontramos os valores

n = 1, 2, 3, . . . , 55,

T (n): 1, 6, 15, . . . , 5995;

T (56) = 6216,

contrariando a validade da sentena aberta dada.

So verdadeiros os resultados (3) e (4). [

Justique!].

12

Introduo Teoria dos Nmeros  [Antonio Carlos & Ana Paula Marques]

2.2
Seja

a Teorema da Induo Finita (1 forma)


com

P (n) uma sentena aberta associada a cada nmero natural n n0 , (i) P (n0 ) verdade; (ii) Para todo n n0 , se P (n) verdade, ento P (n + 1) verdade. Ento P (n) verdade para todo n n0 .
A vericao usa a Boa Ordenao dos nmeros naturais. Seja

n0

natural, tal que:

S = {x N ; x n0 e P (x) falsa.}. Suponhamos, por absurdo, que S = ; ento, pelo princpio da boa ordenao, S possui um menor elemento m; como m S , deve ser m n0 ; pela condio (i) do teorema, temos n0 / S , logo m > n0 e m 1 n0 ; ora, sendo m = min S , segue que m 1 / S , donde P (m 1) verdadeira. Enm, da condio (ii), vem P (m) verdadeira, portanto m / S . Contradio.

EXEMPLO 2.3
1. Mostremos que Alguns termos de

Sn = 1 + 3 + 5 + . . . + (2n 1) = n2 , n 1. Sn sugerem que, realmente, a expresso de Sn = n2 correta...! S1 = 1, S2 = 1 + 3 = 4, S3 = 1 + 3 + 5 = 9, ...

Observemos:

Usando o teorema da induo nita, temos:

S1 = 1 = 12 verdadeira; 2 (ii) Supondo n 1 e Sn = n , caculemos Sn+1 . Temos: Sn+1 = 1 + 3 + . . . + (2n 1) + (2n + 1) = Sn + (2n + 1) = n2 + (2n + 1) = n2 + 2n + 1, 2 isto , Sn+1 = (n + 1) coincide com a expresso enunciada! Segue do teorema que a sentena Sn = 1 + 3 + 5 + . . . + (2n 1) = n2 vale para todo n 1.
(i) 2. Vericar que Pondo

2n > n2 , 2n

para todo

n 5.

P (n) : > P (5) : 25 = 32 > 25 = 52 verdadeira. Supondo n 5 e 2n > n2 , temos, multiplicando a desigualdade por 2, 2 2n > 2 n2 , ou 2n+1 > 2n2 . Enm, 2n2 > (n + 1)2 = n2 + 2n + 1, pois 2n2 n2 2n 1 = n2 2n 1 = (n 1)2 2 > 0 equivale a (n 1)2 > 2, o que evidente, pois n 5.
3. Uma numrica dada por

n2 , temos

Progresso Geomtrica com primeiro termo a1 e razo q (q = 0 e q = 1) uma sequncia

1 qn ; 1q 1 xn+1 2 3 n Em particular, vale 1 + x + x + x + + x = , n 1, x = 1. 1x 0 n1 )q , (a) A expresso dada vale para n = 1, pois q = 1. Supondo n 1, temos an+1 = an q = (a1 q n isto , an+1 = a1 q .
(b) Se

an+1 = an q se n 1 . Por induo sobre n, veriquemos as relaes: n1 , n 1; (a) an = a1 q Sn = a1 + a2 + + an ,


ento

Sn = a1

(b) Para a soma dos termos da PG, comeamos observando que, multiplicando por

a soma, vem:

Sn = a1 + a2 + + an Sn q = a1 q + a2 q + + an q = a2 + a3 + + an + an q . Subtraindo membro 1 qn a1 an q = a1 . a membro, vem Sn Sn q = a1 an q , ou Sn (1 q ) = a1 an q , donde Sn = 1q 1q


Na realidade, zemos um rascunho para melhor perceber a frmula da soma. Ainda necessria uma prova formal por induo! Vamos l. Para supondo

n 1,

temos:

Sn+1 = Sn + an+1

n = 1, tudo bem, pois S1 = a1 . Para a1 an q a1 an+1 q = + an+1 = . 1q 1q

a etapa indutiva, j

Induo Finita Atividade-proposta 2.4


1. Use o teorema da Induo Finita para vericar as relaes.

13

1 1 1 n + + + = , n 1; 12 23 n(n + 1) n+1 3 3 3 2 (b) 1 + 2 + + n = (1 + 2 + + n) , n 1 ; n(n + 1)(n + 2) (c) 1.2 + 2.3 + + n(n + 1) = . 3


(a) 2.[TARSKI] Aponte a falha da seguinte demonstrao.

Proposio Se E um conjunto com n elementos (n 1), ento os elementos de E so todos iguais.

Demonstrao.

n 1 e a armao verdadeira para conjuntos com n elementos. Seja En+1 um conjunto qualquer com n + 1 elementos, x1 , . . . , xn+1 ; como os conjuntos {x1 , x2 , . . . , xn } e {x2 , x3 , . . . , xn+1 } tm n elementos, segue da hiptese que x1 = x2 = . . . = xn e x2 = x3 = . . . = xn+1 , donde a igualdade entre todos os elementos de En+1 . Pelo Teorema da Induo Finita, vale o resultado para todo n 1.
O resultado sendo vlido para

n = 1,

suponhamos

3. Ache uma frmula para cada uma das seguintes somas. (a)

2 + 4 + + 2 n; n (b) 2 + 4 + 8 + + 2
4. Uma vitria rgia encontra-se em um tanque de gua. Sabendo que ela dobra de rea a cada dia, e que, no nal de 20 dias, ela ocupa toda a superfcie do tanque, em qual dia ela ocupar a metade da superfcie do tanque?

2.5

a Teorema da Induo Finita (2 forma)

O mtodo da induo que aplicamos at aqui, parte de uma posio inicial

n0 N e procura validar as expresses subsequentes passando de uma posio para a posio consecutiva [n (n + 1)]. H uma outra forma desse mtodo que, aps a vericao do ndice de partida n0 , procura validar uma
posio supondo vlidas todas as posies anteriores!

Teorema
(i)

Seja

P (n)

uma sentena aberta associada ao nmero natural

n n0 ,

com

n0

natural.

Suponhamos que:

P (n0 )

verdade; se a validade de

(ii) Se ento

n > n0 e, para cada inteiro k , n0 k < n, P (n) verdade para todo n n0 .

P (k ) implicar na validade de P (n),

A demonstrao uma adaptao da induo j vista. Seja Se

S = e m = min S , ento m = n0 ; como m mnimo, n0 k < m. Ento, por (ii), P (m) verdadeira. Contradio.
realidade,

S = {n N ; n n0 e P (n) falsa}. ento vale P (k ) para cada k vericando


para todo

Exemplo Seja a sequncia (an ) dada por a1 = 1, a2 = 3 e an = an1 + an2

n 3.

Na

(an ) uma sequncia denida por recorrncia. Mostrar que an < n 1. 2 Temos a1 = 1 < (7/4), a2 = 3 < (7/4) = (49/16); se n 3 e supondo a relao vlida para todo 1 k < n, vem an = an1 + an2 < (7/4)n1 + (7/4)n2 = (7/4)n2 ((7/4) + 1) = (7/4)n2 (11/4); n2 (49/16) = (7/4)n . enm, an < (7/4)

(1/4)n ,

14

Introduo Teoria dos Nmeros  [Antonio Carlos & Ana Paula Marques]

2.6 Recorrncia
Uma denio por recorrncia para uma certa expresso

En ,

associada ao inteiro

aplicao interessante do mtodo da induo. Por exemplo, basta denir a como obter

n a, uma expresso Ea e mostrar


o

En+1

a partir de

En ,

para todo

auxlio de termos iniciais, digamos

n a. Uma denio semelhante pode ser feita com E1 , E2 , . . . , Er e a informao para obter os termos posteriores.

Exemplo 2.7
1. Para denir a funo fatorial de

n N, f (n) = n!,
isto ,

podemos usar a recorrncia: e

f (0) = 0
Assim, Assim

f (n + 1) = (n + 1)f (n),

0! = 1

(n + 1)! = (n + 1)n!

para todo

n 0.

f (6) = 6 f (5) = 6 5 f (4) = 6 5 4 f (3) = 6 5 4 3 f (2) = 6 5 4 3 2 f (1) = 6 5 4 3 2 1 f (0). 6! = 720. an = an1 + n,


n
onde

2. Consideremos a recorrncia observemos que

a0 = 1.
n

Para calcular uma expresso para

an ,

an an1 = n,

donde, somando,

(aj aj 1 ) =
j =1
Ora, a soma do primeiro membro d 3.

j
j =1

an a0 = an 1,

donde

an = 1 +

[Potncia de nmeros inteiros]


an ,
com

n(n + 1) . 2

Para denir as potncias de um nmero inteiro

n N,

vamos proceder por recorrncia.

1 Pondo a

=a

0 e a

= 1,

se

0, e supondo an denido, tomemos m > 0, n > 0,


temos

an+1 = an a.

Usando induo, podemos vericar as propriedades usuais das potncias. Dados os inteiros (a) (b) (c)

a, b, m, n,

com

am an = am+n ; (am )n = amn ; (a b)n = an bn a


e

Veriquemos (a). Fixando Se

arbitrariamente, usemos induo sobre

n.

am

n= an = am+n ,

1, ento, pela denio dada, temos am


temos que

a1

am

a =

am+1 . Por outro lado, supondo que

am an+1 = am (an a) = (am an ) a = am+n a = am+n+1 ,


donde o resultado, pelo teorema da induo.

Pratique um pouco! Complete a demonstrao de (b) e (c), procedendo como zemos acima.

Atividade-proposta 2.8
1. Considere a sequncia recorrente

an+1 = 2an + 1, n 1,

dado

a1 = 1.
e

Examine alguns termos

para perceber a lei de formao e prove seu prognstico com o teorema da induo. 2. Uma funo Calcule

g : N N

denida recursivamente por

g (1) = 2

g (n) = 2 g(n1)

se

n 2.

g (4). Rn
de regies em que

3. [A pizza de Steiner] Determine o nmero mximo Sugesto. Verique que

retas dividem o plano.

Rn = Rn1 + n, n 1.

Selecionamos, a seguir, trs grupos de aplicaes, que ilustram o uso da metodologia indutiva e alguns resultados surpreendentes: (I) Nmeros binomiais e o Binmio de Newton; (II) Sequncia de Fibonacci; (iii) A Torre de Hanoi.

Induo Finita

15

2.9 Nmeros binomiais Exemplo motivador Suponhamos uma urna contendo 5 bolas distintas, rotuladas como a,b,c,d,e.
Quantos so os agrupamentos de 3 bolas, retiradas sucessivamente e sem reposio? mentos so chamados de Tais agrupa-

arranjos

no oferece maiores diculdades:

3 simples de 5 elementos tomados 3 a 3, notados A5 , cujo clculo A3 5 = 5 4 3 = 60. Olhando melhor a distribuio desses

agupamentos, encontramos 10 formaes bsicas e, para cada uma, 6 permutaes, totalizando os

10 6 = 60

arranjos. A tabela abaixo ilustra a disposio considerada; representamos apenas as 6

permutaes da primeira coluna; escreva as demais, para treinar. abc acb bac bca cab cba Se convencionarmos identicar, em cada coluna, as respectivas permutaes com cada agrupamento bsico da primeira linha, isto , no levar em conta a abd abe acd ace ade bcd bde bce cde

ordenao de cada permutao, nosso quadro


5 3

car, apenas, com 10 agrupamentos, que correspondem aos a partir do conjunto de 5 elementos

{a, b, c, d, e}.
3 C5

Esses

subconjuntos de 3 elementos formados 10 agrupamentos so as combinaes

simples de 5 elementos, tomados 3 a 3 e notados

ou

; nesta segunda notao, so denominados

nmeros binomiais de 5 sobre 3.


Representando por

3! = 1 2 3 = 6
3 = A3 , 3! C5 5

as permutaes de cada coluna, temos, ento, por denio: ou

Mais

A3 543 60 5 = = = 10 3! 6 6 geralmente, partindo de um conjunto com n elementos, para cada 1 k n, indicaremos: n! Ak n = n(n 1)(n 2) (n k + 1) = (n k )! P n = An = n ! = n ( n 1)( n 2) 3 . 2 . 1 n n! n n n Ak k Cn = = n = ; por extenso, escrevemos: 0! = 1, 1! = 1, = =1 k! k !(n k )! 0 n k
3 = C5

Primeiras propriedades
(1) Complementaridade ou simetria:

n nk =

= n k

n k
, onde

(2) Relao de Stiefel:

n1 n1 + k k1

1 k n.

A igualdade (a) segue direto da denio. Quanto a (b), temos:

n1 n1 + k k1
o resultado.

(n 1)! (n 1)! (n 1)! + = k !(n k 1)! (k 1)!(n k )! (n k 1)!(k 1)! n


e

1 1 + k nk

, donde

(3) Dados os inteiros

k,

com

0 k n,

o coeciente binomial

n k

um nmero inteiro.

De fato, por induo sobre

n,

o resultado evidente para

n = 1.

Supondo vlido pra

n,

temos

n+1 0

n+1 n+1

= 1,

logo inteiros; se

1 k n,

segue da relao de Stifel

n+1 k

n k1

n k , logo

inteiro, pela hiptese de induo.

16

Introduo Teoria dos Nmeros  [Antonio Carlos & Ana Paula Marques]

2.10 O tringulo de Pascal


Vamos dispor os nmeros binomiais em linhas e colunas, como indicado a seguir, usando sistematicamente a relao de Siefel.

n 0 1 2 3 4 5 6 7 8 9 10

n 0

n 1

n 2

n 3

n 4

n 5

n 6

n 7

n 8

n 9

n 10

1 1 1 1 1 1 1 1 1 1 1

1 2 3 4 5 6 7 8 9 10

1 3 6 10 15 21 28 36 45

1 4 1 10 5 1 20 15 6 1 35 35 21 7 1 56 70 56 28 8 84 126 126 84 36 120 210 252 210 120

1 9 45

1 10

Como veremos a seguir, h um grande nmero de propriedades binomiais escondidas no tringulo.

2.11 O binmio de Newton


Considerando sucessivas potncias da soma

a+b

de dois nmeros inteiros:

(a + b)1 (a + b)2 (a + b)3 (a + b)4

=a+b = a2 + 2ab + b2 = a3 + 3a2 b + 3ab2 + b3 = a4 + 4a3 b + 6a2 b2 + 4ab3 + b4

observamos que os coecientes dos segundos membros correspondem s linhas do tringulo de Pascal (veja acima!). Tal fato tambm no passou desapercebido por Newton que sistematizou as potncias, inclusive generalizando para o caso de expoente no-inteiro,o que fornece (sua) srie binomial!!

Teorema Dados

inteiros e

n N ,
n 1

temos

(a + b)n = an +
A demonstrao, por induo sobre

an1 b +

n 2

an2 b2 + +

n n1

abn1 + bn n=1
sendo

n,

usa, essencialmente, a relao de Stiefel. O caso

evidente, suponhamos a expresso vlida para

n1

e examinemos o expoente

n + 1:

(a + b)n+1 = (a + b)(a + b)n = a (a + b)n + b (a + b)n ; calculando cada n n1 2 n n 2 n1 + n abn a (a + b)n = an+1 + n b + + n 1 a b+ 2 a 1 a b n b (a + b)n = an b + (a + b)n+1 .
n 1

parcela:

an1 b2 + +

n n2

a2 bn1 +

n n1

abn + bn+1

Somando membro a membro as duas ltimas igualdades e usando a relao de Stiefel, vemos que vale

Corolrio 1
Basta fazer

n k=0

n k

= 2n
no binmio. Em particular, recuperamos o bem conhecido resultado sobre o

a=b=1

nmero de subconjuntos um conjunto de Pascal.

elementos. Observe a soma de cada linha do tringulo de

Corolrio 2

(1)k
k=0

n k

= 0.

De fato, escolhamos

a=1

b = 1

no binmio.

Induo Finita Atividade-proposta 2.12

17

1. Estabelecer a identidade das diagonais (depois de uma anlise cuidadosa do tringulo de Pascal):

m m+1 m+2 m+n + + + + 0 1 2 n m m+1 n + + + = m m m n n k+1 S= k (k + 1) = 2 2


k=1 k=1

m+n+1 n

2. Vericar a identidade das colunas (depois de uma anlise cuidadosa do tringulo de Pascal):

n+1 m+1
, ento

Aplicao Se

S=2
,

n+2 3

= n(n + 1)(n + 2)/3.

3.

[Absoro] Verique a igualdade k


n
(a)

n k
n

=n

n1 k1

1 k n.

4. Verique as igualdades:

k=1
5. Mostrar que, se

n k k m
e

= n2 n

n1

(b)

k=0

n 1 k+1 k

1 2n+1 1 n+1

so inteiros positivos, ento

(m + n)! m! n! m+n < mm nn (m + n)

2.13 Sequncia de Fibonacci


A

sequncia de Fibonacci

uma sequncia de nmeros naturais, denida por recorrncia pelas

relaes:

f1 = f2 = 1; fn = fn1 + fn2 , n 3.
Tais relaes permitem obter os termos

f = {1, 1, 2, 3, 5, 8, 13, 21, 34, 55, 89, 144, 233, . . .}

Possuindo notveis propriedades aritmticas, a sequncia possui inmeras interpretaes, dentre as quais a de Leonardo de Pisa, seu grande estudioso: Um casal de coelhos recm-nascidos foi posto em um local cercado. A cada ms, um casal de coelhos produz outro casal; um casal comea a procriar dois meses aps o seu nascimento. Quantos casais estaro presentes aps um ano? Seguindo o modelo de procriao enunciado, temos a tabela: ms 1 2 3 4 5 6 7 8 9 10 11 12 n casais ms anterior 0 1 1 2 3 5 8 13 21 n casais recm-nasc 1 0 1 1 2 3 5 8 13 total 1 1 2 3 5 8 13 21 34

Pratique um pouco!

Complete os dados ausentes da tabela.

18

Introduo Teoria dos Nmeros  [Antonio Carlos & Ana Paula Marques]

Frmula de Binet Essa relao permite achar o termo geral da sequncia, sem recorrer aos termos
anteriores. A idia original encontrar progresses geomtricas

n n1 + q n2 , u n = un1 + un 2 , donde q = q 1+ 5 1 5 n n raizes so = e = . Assim, fn = a + b satisfaz a recorrncia, desde f1 = 1 2 2 f2 = 1, o que permite achar a e b: a = 1/ 5 e b = 1/ 5, donde, a expresso procurada: n n 1 5 1+ 5 2 2 n n fn = = 5 5
recorrncia dada:

un = q n , q = 0, que vericam a 2 2 ou q = q + 1 = 0, q q 1 = 0, cujas


e

Observemos as simplicaes que devem ocorrer com todas essas raizes de 5, para resultar gentis nmeros inteiros!

EXEMPLO 2.14
1. Veriquemos que

f1 + f2 + + fn = fn+2 1.

Temos as igualdades sucessivas:

f1 = f3 f2 f2 = f4 f3 f3 = f5 f4 fn1 = fn+1 fn fn = fn+2 fn+1


Somando membro a membro, segue 2. Dados

f1 + f2 + + fn = fn+2 2, fn+m = fn1 fm + fn fm+1 .

donde o resultado, pois

f2 = 1.

n 2, m 1,

vericar que

m (n estando arbitrariamente xado). Para m=1, temos fn+1 = fn1 f1 + fn f2 = fn1 + fn , relao vlida. Suponhamos, a seguir, a m = 1, 2, . . . , k e veriquemos que ainda vale para m = k + 1. Temos fn+k = fn1 fk + fn fk+1 fn+(k1) = fn1 fk1 + fn fk fn+k + fn+(k1) = fn1 (fk + fk1 ) + fn (fk+1 + fk ) fn+(k+1) = fn1 fk+1 + fn fk+2
Usemos induo sobre . Essa ltima expresso comprova a validade. 3. Para comprovar que

relao vlida para

2 + f2 fn n+1 = f2n+1 , a frmula de Binet mais signicativa: 1 2n 1 n n 2 ( + 2n 2(1)n ); analogamente, encontramos partindo de fn = ( ), vem fn = 5 5 1 2n+2 1 2 2 n 2 + f2 2n+1 + 2n+1 ) = f fn+1 = ( + +2 2(1)n+1 ). Enm, fn 2n+1 n+1 = ( 5 5

Atividade-proposta 2.15
1. Seja (a)

(fn )

a sequncia de Fibonacci; verique que: (b)

f2 + f4 + + f2n = f2n+1 1 A= 1 1 1 0
. Mostre que

2 + f2 + + f2 = f f f1 n n+1 n 2

2. Seja

An =

fn+1 fn fn fn1

n 2.

Usando determinantes na relao anterior, obter a identidade de Cassini:

2 = (1)n , n 2. fn1 fn+1 fn

Induo Finita [Lucas] Seja dn =


n1 0 n2 1 n3 2

19

3.

+ =
k

n1k k dn = fn .

uma diagonal ascendente

do tringulo de Pascal, onde

k=

n1 2

. Mostre que

2.16 Torre de Hanoi


Trata-se de um jogo idealizado pelo matemtico francs Edouard Lucas, em 1882. Consiste de

discos de dimetros diferentes com um furo no seu centro, e uma base onde esto ncadas trs hastes. Inicialmente, os discos esto enados numa haste de modo que nenhum disco esteja sobre um outro de dimetro menor. O objetivo mover todos os discos, da haste original a uma outra, prdeterminada, com as seguintes regras: a) somente um disco pode ser movido de cada vez; b) um disco maior nunca pode ser posto sobre um disco menor. Seja

T (n) o nmero mnimo para a trsnferncia dos discos. T (1) = 1, T (2) = 3, T (3) = 7, T (4) = 15... Mas qual seria T (n).

Brincando um pouco, no difcil ver que a forma genrica de

Podemos usar um raciocnio indutivo: supondo conhecido um processo para saberemos obter A idia muito simples: transferimos

T (n)? obter T (n 1),

ento

(n 1)

discos de uma pilha par outra,

deixando o maior disco em sua haste inicial. Feito isso, deslocamos o disco maior para sua haste nal, o que libera a haste inicial. Enm, trazemos os quantos movimentos para transferir os Assim, obtemos

n1

discos sobre o disco maior. Portanto, zemos

n discos? s contar: T (n 1) + 1 + T (n 1) = 1 + 2T (n 1). T (n) = 1 + 2T (n 1), com n 2, e T (1) = 1. Ora, na Atividade-proposta 2.8,
tenha sido obtido!!

exerccio 1, solicitamos uma frmula para a sequncia recorrente acima. Esperamos que o valor correto

T (n) = 2n 1, n 1,

Uma lenda apocalptica acompanha o jogo. No incio dos tempos, sacerdotes de um templo oriental deveriam transferir 64 discos de ouro puro ao redor de 3 colunas de diamante. Quando os 64 discos fossem transferidos, o mundo acabaria...! Pois bem, o nmero mnimo para transferirmos 64 discos

T = 264 1,

que um nmero meio grande.

Se cada movimento durar 1 segundo, como em

cada sculo, h 3.110.400.000 segundos, verique que sero necessrios transferncia!!

5, 93067 109

sculos para a

Divisibilidade e Algoritmo Euclidiano da Diviso

23

CAPTULO 3 DIVISIBILIDADE ALGORITMO EUCLIDIANO DA DIVISO


No h dvida sobre o surgimento de smbolos pictogrcos expressando numerais abstratos. Tambm est bem rmado (em cerca de 3100AC) que a contagem abstrata precursora da origem da escrita. DAMEROW, P. The material culture of calculation. Berlin: Max Planck Institut, 1999, p.12 et passim.

Objetivos do Captulo 3

(a) Estabelecer relao de divisibilidade de nmeros inteiros; (b) Descrever o algoritmo da diviso euclidiana (existncia e unicidade); (c) Explicitar as propriedades da diviso de inteiros; (d) Estabecer a construo de sistemas de numerao, com nfase no sistema binrio e suas aplicaes; (e) Descrever o algorimo de Euclides para o clculo do mdc; (f) Explicitar as propriedades do mmc; (g) Aplicar a teoria a problemas elementares de contagem.

3.1

INTRODUO

Precursor do mtodo axiomtico, Euclides de Alexandria (c.330-275 AC) foi o primeiro matemtico a tratar a Geometria e a Aritmtica como cincias dedutivas. Os Elementos, uma coleo de 13 livros, apresentou os fatos matemticos de sua poca com notvel rigor, descrevendo cada teoria a partir de denies, postulados e axiomas, e seu ordenamento em teoremas, cujas demonstraes seguem regras lgicas bem determinadas. A Aritmtica tem incio no livro VII, onde, dentre inmeros resultados, usada sistematicamente a chamada Diviso Euclidiana, algumas propriedades dos nmeros primos (inclusive sua demonstrao sobre a innidade de primos) e o algoritmo para o clculo efetivo do mdc de dois inteiros. Os Elementos so (a seguir Bblia), provavelmente, o livro mais reproduzido e estudado na histria do mundo ocidental.

24

Introduo Teoria dos Nmeros  [Antonio Carlos & Ana Paula Marques]

3.2

DIVISIBILIDADE em

Dados dois nmeros inteiros a e b, com a = 0, diremos que a divide b quando existir c Z tal que b = a c; neste caso, diremos tambm que a um divisor de b, ou a um fator de b ou, ainda, que b um mltiplo de a. Usaremos a notao a|b para indicar que a divide b e sua negao, por a | b. Observemos que a notao a|b nada tem a ver com fraes. Por exemplo, 2|0, 1|10, 4|4, 2|4; 3 | 4; 5 | 2; 5 | 11.
b b = ac = ad, como a = 0, vem que c = d. O inteiro c o quociente de b por a e denotado por c = . a 0 10 4 4 Por exemplo, = 0, = 10, = 1; = 2. 2 1 4 2

Se a|b (a = 0), ento o inteiro c tal que b = a c unicamente determinado por essa condio, pois se

3.3 Propriedades

(1) Sejam a, b Z e c Z. Temos: (i) 1|c; a|a e a|0; (ii) se a|b e b|c, ento a|c
O item (i) decorre das igualdades c = 1 c, a = a 1 e a 0 = 0. No caso (ii), por hiptese, existem x, y Z tais que b = ax e c = by , donde c = by = (ax)y = a(xy ), ou a|c. Em particular, todo nmero inteiro divisvel por 1 e, se no nulo, por si mesmo.

(2) Sejam a, b Z ; se a|b, ento |a| |b|. Como existe um inteiro c tal que b = ac, a hiptese b = 0 exige c = 0, logo |c| 1, donde |a| |c| |a|, isto , |b| |a|. Em particular, se a|1, ento |a| 1, donde |a| = 1, ou a = 1. (3) Em N a divisibilidade uma relao de ordem, isto , possui as propriedades: (i) reexiva: a|a; (ii) transitiva: a|b, b|c a|c; (iii) anti-simtrica: a|b, b|a a = b. De fato, j vimos a validade de (i) e (ii) em Z . Quanto antisimetria, se a, b, x, y so inteiros no nulos e vericam a = bx e b = ay , ento b = b(xy ), donde xy = 1, logo, em N , x = y = 1 e a = b. (4) Sejam os inteiros a, b, c, com c = 0. Se c|a e c|b, ento, para todos os inteiros e , vale c|(a + b). De fato, se a = cx e b = cy , ento a + b = (cx) + (cy ) = (x + y )c, donde o resultado. Observao Dados os inteiros a, b, c, com c = 0, a condio c|(a + b) no acarreta c|a ou c|b. Por exemplo, claro que 2|(7 + 3), mas 2 | 7 nem 2 | 3. Vale, entretanto, o seguinte resultado: se a = 0, a|(b + c) e a|b, ento a|c. De fato, se b + c = xa e b = ya, ento ya + c = xa, logo xa ya = c, isto , (x y )a = c, donde a|c. (5) Sejam os inteiros a, b, c, d, com a = 0 e c = 0. Se a|b e c|d, ento ac|bd.
Temos b = xa e d = cy , logo (bd) = (xy )(ac). Em particular, se a|b, ento ac|bc, para todo c Z .

Divisibilidade e Algoritmo Euclidiano da Diviso 3.4 Proposio


Se a = 0 e b = 0 so inteiros, e n 0 um nmero natural, veriquemos as seguintes relaes: (a) (a b)|(an bn ) e an bn = (a b) (an1 + an2 b + + abn2 + bn1 ); (b) (a + b)|(a2n+1 + b2n+1 ) e a2n+1 + b2n+1 = (a + b)(a2n a2n1 b + ab2n1 + b2n (c) (a + b)|(a2n b2n ) e a2n b2n = (a + b)(a2n1 a2n2 b + + ab2n2 b2n1

25

Observemos que a proposio solicita, em cada item, dois procedimentos: inicialmente, h que vericar a divisibilidade solicitada; em seguida, exibir o quociente. A idia usar induo sobre n, em todos os casos. (a) A armao verdade para n = 0, pois a b divide a0 b0 = 0. Suponhamos, agora, que a b|an bn ; escrevendo an+1 bn+1 = aan ban + ban bbn = (a b)an + b(an bn ), vemos que a b|an+1 bn+1 , usando a propriedade (4) do item anterior. (b) Procedendo analogamente, a etapa indutiva resulta de
a2(n+1)+1 + b2(n+1)+1 = a2 a2n+1 b2 a2n+1 + b2 a2n+1 b2 b2n+1 = (a2 b2 )a2n+1 + b2 (a2n+1 b2n+1 ).

(c) Enm, no caso (c), a etapa indutiva decorre das relaes:


a2(n+1) + b2(n+1) = a2 a2n b2 a2n + b2 a2n b2 b2n = (a2 b2 )a2n + b2 (a2n b2n ).

EXEMPLO 3.5
1. Se a e b so inteiros positivos para os quais + um inteiro, ento a = b. Alm disso, mostre a b que a = 1 ou a = 2.
1 1 a+b + = mostra que deve ser ab|a + b; segue,ento, que a|a + b, donde a|b; a b ab 1 1 2 analogamente, b|a. Assim, concluimos que a = b. Voltando primeira condio, temos + = , a a a um inteiro, por hiptese, donde a|2 e a = 2 ou a = 1. 1 1

De fato, a expresso

2. Ache todos os inteiros n 1 para os quais (n + 1)|(n2 + 1). Como n2 + 1 = [(n + 1) 1]2 + 1 = (n + 1)2 2(n + 1) + 2, vemos que (n + 1)|(n2 + 1) (n + 1)|2, donde n + 1 = 1 (o que impossvel, pois n > 0) ou n + 1 = 2, donde n = 1. 3. Para todo n 1, mostre que 3|22n 1. Podemos escrever: 22n 1 = (22 )n 1 = 4n 1 = (3 + 1)n 1 = 3n + isto , um mltiplo de 3. 4. Usando induo sobre n 1, vercar que 8|32n + 7. A relao vale se n = 1, pois 32 + 7 = 9 + 7 = 16 mltiplo de 8. Suponhamos n 1 e a relao vlida para n, isto , 8|32n + 7, ou 32n + 7 = 8x. Podemos escrever 32n+2 + 7 = 9(32n ) + 7 = 9(8x 7) + 7 = 72x 56, que bem um mltiplo de 8. 5. Use as relaes polinomiais a b|an bn , a + b|a2n+1 + b2n+1 e a + b|a2n b2n para mostrar que (a) 9|10n 1; (b) 8|32n 1; (c) 53|74n 24n ; (d) 6|52n+1 + 1 Temos: (a) 9 = 10 1|10n 1n = 10n 1; (b) 32n 1 = (32 )n 1 = 9n 1, logo 8 = 9 1|9n 1; (c) 74n 24n = (72 )2n (22 )2n = 492n 42n mltiplo de 49 + 4 = 53; (d) 6 = 5 + 1|52n+1 + 1.
n 1

3n1 + +

n n1

3 + 1 1,

26

Introduo Teoria dos Nmeros  [Antonio Carlos & Ana Paula Marques]

6. Considere a sequncia de Fibonacci (fn ). Dados 1 m < n, se m|n ento fm | fn . Sugesto. Use a relao de (2.14-(2)), e verique que fm |fqm . Se m|n, seja q N tal que n = mq ; para mostrar que fm |fqm , usaremos induo sobre q 1, e a relao fn+m = fn1 fm + fn fm+1 . O resultado sendo imediato pra q = 1, consideremos, sucessivamente, n = m, n = 2m, , n = (q 1)m na relao anterior:
fm1 fm + fm fm+1 = f2m fm |f2m f2m1 fm + f2m fm+1 = f3m fm |f3m

. . .

f(q1)m1 fm + f(q1)m fm+1 = fqm fm |fqm

Atividade-proposta 3.6
1. Se possvel, d exemplos de nmeros inteiros a, b, c, com a = 0, a < b e a < c, tais que a | bc, com a |b e a |c. 2. Seja X um nmero natural de 2 dgitos e Y o nmero obtido de X quando so permutados seus (dois) dgitos. Mostre que 9|Y X e ache todos os inteiros X tais que |Y X | = 18. 3. Para todo n 1, use o binmio de Newton para mostrar que n2 |(n + 1)n 1 . 4. Mostre que 147 + 247 + 347 + 447 + 547 + 647 mltiplo de 7. 5. Fixados a 2 e 0 < m n, mostre que am |an . Alm disso, se m|n, ento am 1|an 1. 6. Seja Nn um inteiro formado de n 1's consecutivos. Por exemplo, N3 = 111,N7 = 1111111. Se 1 m < n e m | n, ento Nm | Nn .

Diviso Euclidiana 3.7 Exemplo motivador Para comparar (as medidas de) dois segmentos, associados aos nmeros naturais a e b, e tomando b como referncia, podemos considerar os sucessivos segmentos b, 2b, 3b, , qb, onde, se a no mltiplo de b, o segmento qb o menor segmento que no ultrapassa a, isto , qb < a < (q + 1)b. Observemos que a existncia de q est garantida pela propriedade arquimediana [cf. 1.7  Prop. C]. Assim, pondo r = a qb, temos a = qb + r, com 0 r < b.
0
r b r 2b

qb r r (q+1)b r a

A interpretao acima, na realidade, um poderoso algoritmo, denominado de Diviso Euclidiana, que passaremos a considerar em toda sua aplicabilidade.

Teorema Dados a, b Z, com b = 0, existem inteiros q e r tais que a = bq + r, onde 0 r < |b|. Alm disso, os inteiros q e r so unicamente determinados pelas condies anteriores.
Faremos a demonstrao em vrias etapas. 10 caso: b > 0. Seja S = {x N ; x = a bn, para algum n Z}. Notemos que S no vazio, pois para n = |a|, temos a bx = a + b|a| a + |a| 0; pela boa ordem de N, existe r = min S e temos r 0 e r = a bq , isto , a = bq + r, onde q Z. Se, por absurdo, b r, temos r = b + s, onde s 0 e como b > 0 teremos s < r, portanto s / S ; mas, por outro lado, s = r b = a bq b = a b(q + 1), logo s S e chegamos a uma contradio!.

Divisibilidade e Algoritmo Euclidiano da Diviso

27

20 caso: b < 0. Pelo caso anterior, existem inteiros q1 , r1 tais que a = q1 (b) + r1 = b(q1 ) + r1 , onde 0 r1 < b = |b|. Portanto, basta escolher q = q1 e r = r1 . 30 caso. Unicidade do par (q,r). De fato, se (q1 , r1 ) tambm verica a = bq + r = b1 q + r1 , com 0 r |b| e 0 r1 |b|, teremos b(q q1 ) = r1 r, logo |b||q q1 | = |r1 r|. Se r = r1 , teremos |q q1 | 1, logo |b|(q q1 ) |b|, |r1 r| |b| ; por outro lado, |r1 r| < |b|. Contradio!

Nas condies do teorema acima, os nmeros q e r so, respectivamente, denominados quociente e resto da diviso euclidiana de a por b; observemos que b|a se, e s se, nulo o resto r = 0.

EXEMPLO 3.8
1. Vamos calcular o quociente e o resto da diviso de a = 680 por b = 12 (4 casos!). a) O caso mais simples (a = 680, b = 12) j nosso velho conhecido: 680 = 12 56 + 8; b) Se a = 680 e b = 12, a expresso anterior nos d: 680 = (12) (56) + 8, onde q = 56 e r = 8 < |b| = 12; c) Com a = 680 e b = 12, vem 680 = 12 (56) 8. Ateno! Essa igualdade no representa uma diviso euclidiana, pois o candidato a resto negativo! Para contornar essa diculdade, vamos somar e subtrair b = 12 ao segundo membro: 680 = 12 (56) 8 12 + 12 = 12 (57) + 4; observe que r = 4 < b = 12 d) O ltimo caso a = 680 e b = 12 pode ser formado de (c): 680 = 57(12) + 4. 2. Para cada x R, a funo maior inteiro contido em x dada por: se n Z e n x < n + 1, ento I (x) = [x] = n. Se b > 0 um inteiro, consideremos a diviso euclidiana a = qb + r, 0 r < b. Temos q = De fato, temos
a b

=q+r b , onde

b b a Aplicao Se 0 < b < a, ento h q = mltiplos no nulos de b menores ou iguais a a. Por b exemplo, entre 1 e 1000, h [1000/7] = 142 inteiros divisveis por 7.

um nmero racional tal que 0 r b < 1. Segue, ento, a r a q = q + < q + 1, logo q = .


r b

a . b

3. Fixado um nmero natural m 2, todo inteiro n se escreve, de modo nico, na forma n = mq + r, com 0 r < m, isto , o resto pode assumir m possveis valores distintos r = 0, 1, 2, , (m 1); assim, os inteiros se dividem em m classes, cada uma com o valor de r correspondente. Por exemplo, (a) m = 2; h 2 classes de inteiros: os da forma 2q (inteiros pares) e os da forma 2q + 1 (mpares); (b) m = 3; h 3 classes: inteiros da forma 3k, ou 3k + 1 ou 3k + 2; (c) m = 4; h 4 classes: inteiros das formas 4q , 4q + 1, 4q + 2, 4q + 3. 4. Vericar que o quadrado de um inteiro da forma 4k ou 4k + 1. Basta observar cada uma das quatro classes do resto (diviso por 4): a = 4q a2 = 16q 2 = 4k; a = 4q + 1 a2 = (4q + 1)2 = 16q 2 + 8q + 1 = 4K + 1; a = 4q + 2 a2 = (4q + 2)2 = 16q 2 + 16q + 4 = 4Q; a = 4q + 3 a2 = (4q + 3)2 = 16q 2 + 24q + 9 = 4t + 1. Em particular, todo quadrado perfeito impar da forma 4k + 1.

Pratique um pouco! Mostre que nenhum termo da sequncia 11, 111, 1111, ... um quadrado perfeito.
5. Verique as relaes, onde a Z: (a) a par a2 par; (b) a impar a2 impar.

28

Introduo Teoria dos Nmeros  [Antonio Carlos & Ana Paula Marques]

Se a = 2q par, ento a2 = 4q 2 = 2k par. Analogamente, se a = 2m + 1 impar, ento a2 = 4m2 + 4m + 1 = 2q + 1 impar. Reciprocamente, se a2 par, ento no pode ser a impar, do contrrio, seria a2 impar, contra a hiptese; ora, se no pode ser a impar, porque a par, pois h exatamente dois tipos disjuntos de classes de inteiros: a dos nmeros pares e a dos nmeros mpares. Analogamente, se a2 impar, ento a impar.

Atividade-proposta 3.9
1. Em uma diviso de nmeros naturais, o dividendo vale 802 e o quociente 14; encontre os possveis divisores e restos. 2. Mostre como, usando uma calculadora que s realiza as quatro operaes, pode-se efetuar a diviso euclidiana de dois nmeros naturais em apenas trs passos. Aplique esse processo para dividir a = 12.615 por b = 58. 3. Determine quantos dos inteiros x tais que 1 x 1000 so divisveis por 4, e quantos, vericando 100 x 999 so divisveis por 5. 4. Use o algoritmo da diviso para vericar que: (a) Todo inteiro mpar da forma 4k + 1 ou 4k + 3; (b) O quadrado de um inteiro da forma 3k ou 3k + 1; (c) O cubo de um inteiro da forma 9k, 9k + 1 ou 9k + 8. 5. Se o inteiro a no divisvel por 3, ento a2 deixa resto 1 na diviso por 3. Conclua que, se 3|a2 + b2 , ento a e b so divisveis por 3. 6. Mostre que o produto de k nmeros naturais consecutivos divisvel por k!. Conclua que, para todo n N: (i) 6|n(n + 1)(n + 2); (ii) 6|n(n + 1)(2n + 1).

3.10 Sistemas de numerao


O algoritmo da diviso fornece um mtodo muito simples para a representao de inteiros em outras bases de numerao que no a base decimal, com a qual estamos j acostumados. Com a expanso do clculo numrico efetivo em computadores digitais, outras bases de numerao voltaram a ser mais estudadas, como a base binria (b = 2) e a hexadecimal (b = 16). O processo est resumido na prxima proposio.

Proposio Dados os inteiros a e b, com a 0 e b > 1, existem inteiros r0 , r1 , . . . , rn , . . ., univocamente determinados pelas condies: (a) Existe um nmero natural m tal que rn = 0 para todo n m; (b) Para todo n, temos 0 rn < b; (c) a = r0 + r1 b + + rn bn + .
Aplicando sucessivamente a diviso euclidiana:
a = bq0 + r0 , r0 < b, q0 = bq1 + r1 , r1 < b, q1 = bq2 + r2 , r2 < b,

e assim por diante. Como a > q0 > q1 > , deve ocorrer, em alguma posio, qn1 < b, logo, da igualdade qn1 = bqn + rn , resulta qn = 0, o que acarreta 0 = qn = qn+1 = , donde 0 = rn+1 = rn+2 = .

Divisibilidade e Algoritmo Euclidiano da Diviso


Obtemos,ento,
a = r0 + r1 b + + rn bn , ou a = rn bn + + r1 b + r0

29

Essa expresso a expanso b-dica do inteiro a. Para representar os nmeros de 0 a b 1, usamos os smbolos S = {s0 , s1 , . . . , sb1 }, onde s0 = 0.

Casos particulares
(a) Sistema binrio: S = {0, 1} (dgitos binrios). Por exemplo, 1002 = 22 = 4, 10112 = 1 + 2 + 23 = 11. (b) Sistema hexadecimal: S = {0, 1, 2, 3, 4, 5, 6, 7, 8, 9, A, B, C, D, E, F }. Por exemplo, 2A716 = 2 162 + 10 16 + 7 = 512 + 160 + 7 = 679.

Exemplos 3.11
1. Na tabela abaixo, verique cada representao: Base 2 Base 10 Base 16 0 0 0 1 1 1 10 2 2 11 3 3 100 4 4 101 5 5 110 6 6 111 7 7 1000 8 8 1001 9 9 1010 10 A 1011 11 B 1100 12 C 1101 13 D 1110 14 E 1111 15 F 10000 16 10 10001 17 11 10100 20 14 1001101 77 4D 2. Quantos algarismos so usados para numerar as 300 pginas de um livro {1, 2, 3, . . . , 300}? Ora, os nmeros de 1 a 9 usam 9 algarismos; os de 10 a 99, usam 2 90 = 180 algarismos; enm, para numerar as pginas de 100 a 300, necessitaremos 3 201 = 603 algarismos. Ao todo, so 9+180+603=792 algarismos. 3. Considere um produto nito de nmeros naturais a1 a2 ak . Vericar que o algarismo das unidades do produto a1 ak igual ao algarismo das unidades do produto dos algarismos da unidades de cada fator. Conclua, ento, que (a) o algarismo das unidades de um quadrado perfeito s pode ser 0,1,4,5,6 ou 9; (b) todo nmero da forma M = n(n + 1)/2 (n N ) inteiro e seu algarismo das unidades no pode ser 2,4,7,nem 9.

30

Introduo Teoria dos Nmeros  [Antonio Carlos & Ana Paula Marques]

Sendo u1 , u2 , . . . uk os respectivos algarismos das unidades dos nmeros dados, temos a1 = m1 10+ u1 , a2 = m2 10 + u2 , logo a1 a2 = m12 10 + u12 , onde mi 10 indica um mltiplo de 10 e ui dgito de unidades (0 ui 9); mais geralmente, vale a1 ak = m1k 10 + u1k . (a) Em particular, se a = m 10 + u, ento o algarismo das unidades de a2 o mesmo de de u2 , isto , 0,1,4,5,6 ou 9. (b) Como n e n+1 so inteiros consecutivos, um deles par, logo n(n+1) par, donde M bem inteiro. O algarismo das unidades de 2M no pode ser 4 ou 8, pois se n termina em {0, 1, 2, 3, 4, 5, 6, 7, 8, 9}, ento n(n + 1) termina em {0, 2, 6, 2, 0, 0, 2, 6, 2, 0}. 4. Em um sistema de numerao de base b > 2, o nmero 121 um quadrado; se for b > 3, ento 1331 um cubo. De fato, 121 = b2 + 2b + 1 = (b + 1)2 ; tambm, 1331 = b3 + 3b2 + 3b + 1 = (b + 1)3 .

Atividade-proposta 3.12
1.Considere um sistema de numerao de base arbitrria b > 0. Mostre que o nmero 10101 divisvel por 111, e escreva o quociente correspondente. 2. No sistema decimal, um nmero se escreve 12551. Encontre a base na qual o nmero da forma 30407. 3. Utilizando os sistemas decimal e binrio, justique o seguinte algoritmo utilizado pelos antigos egpcios para efetuar a multiplicao de dois inteiros positivos a e b representados no sistema decimal. Ponha a e b no alto de duas colunas. Abaixo de a ponha o quociente q0 da diviso de a por 2, abaixo de q0 ponha o quociente q1 da diviso de q0 por 2, etc. Abaixo de b ponha 2b , abaixo de 2b ponha 4b , etc. Toda vez que o nmero da coluna do a for mpar, coloque um sinal + ao lado do nmero da mesma linha da coluna do b . Some todos os nmeros assinalados com o sinal +, este o produto de a por b . Por exemplo, a = 35 e b = 47 : a=35 17 8 4 2 1 b=47 + 94 + 188 376 752 1504 + 1645 = 35 47 Sugesto. Na representao binria a = r0 + r1 2 + + rn 2n , observe que cada resto ri+1 = 1 se o quociente q i impar, e ri+1 = 0 se q i par. Logo, no produto ab = r0 b + r1 2b1 + + rn 2n b, so retidos os termos provenientes dos elementos mpares da coluna da esquerda. 2.[Jogo do NIM] Trata-se de um antigo jogo chins de palitos jogado por duas pessoas. Na realidade, h uma estratgia que, se adotada pelo jogador que inicia o jogo, ele sempre ganhar. Grupos de palitos so alinhados em uma mesa, com um nmero qualquer de palitos e de grupos (colunas). Um movimento consiste em retirar um nmero qualquer de palitos de um grupo, inclusive todo o grupo, mas apenas um grupo pode ser alterado. Os jogadores se alternam e quem retirar o ltimo palito ganha o jogo. Vamos estabelecer uma estratgia de tal modo que, quem iniciar a partida, jogando com a boa regra, sempre vencer!

Divisibilidade e Algoritmo Euclidiano da Diviso

31

Basta escrever o nmero de palitos de cada linha na base 2, e colocando-os de modo que os algarismos das unidades se correspondam. Por exemplo, no incio da partida, temos 15 palitos separados em trs grupos de 3, 5 e 7 palitos: [3] Grupo 1 [5] Grupo 2 1 [7] Grupo 3 1 2 1 0 1 2 1 1 1 3 [3] Grupo 1 [5] Grupo 2 1 [6] Grupo 3 1 2 1 0 1 2 1 1 0 2

Somando os trs nmeros como se fosse na base dez, obtemos 223, a chamada chave do jogo. O primeiro jogador poder, ento, com uma jogada, tornar pares todos os algarismos da chave (veja acima, aps a retirada de um palito do grupo 3). Por sua vez, o segundo jogador transformar a chave 222 numa outra com, pelo menos, um algarismo mpar, o que, mediante uma jogada conveniente, poder ser recolocada numa situao par, a chamada posio segura (para o primeiro jogador), pois ganhadora!
Pratique um pouco!

(a) So seguras as conguraes (2, 2), (1, 2, 3), (2, 3, 6, 7), (1, 2n, 2n + 1), (n, 7 n, 7); (b) No so seguras: (1, 3, 4), (1, 2n + 1, 2n + 2); (c) Transfome cada congurao (1, 3, 9, 27) , (3, 5, 7, 8, 11) numa congurao segura. A soluo nica? (d) Classique as conguraes (1, 1, 1) e (1, 0, 0).

3.13 Mximo Divisor Comum


Sejam a e b inteiros no ambos nulos; um divisor comum de a e b um inteiro c = 0 tal que c|a e c|b. Estamos interessados em explicitar o mximo divisor comum de a e b, como um inteiro d que seja o maior divisor comum de a e b; na realidade, usaremos a seguinte denio. Um inteiro d um mximo divisor comum (mdc) de a e b quando: (i) d um divisor comum de a e b; (ii) se c um divisor comum de a e b, ento c|d. Observemos logo que, se d e D, so inteiros no nulos vericando (i) e (ii), ento d|D e D|d, isto , d = D. Assim, quando existe d = mdc (a, b) = (a, b), se convencionarmos escolher d > 0, ento poderemos especicar o mdc como o nico inteiro positivo vericando (i) e (ii). Vemos, tambm, que, se c = 0 um divisor comum de a, b, ento c|d, donde c |c| d, isto , o mdc o maior dos divisores comuns. Por exemplo, (1) se a Z, (1, a) = 1; (2) se a Z , ento (0, a) = |a|; (a, a) = |a|; (3) se a Z , ento a|b (a, b) = |a|; (4) se a, b Z, no ambos nulos, ento (a, b) = (|a|, |b|).
3.14 Lema de Euclides

Sejam os inteiros a, b, m, n. Se existe (a mb, b) ento (a, b) existe e (a, b) = (a mb, b). Se existe (a, b na) ento (a, b) existe e (a, b) = (a, b na). Pondo, por exemplo, d = (a mb, b), como d|(a mb) e d|b, vemos que d divide a = (a mb) + mb, ou seja, d um divisor comum de a e b. Por outro lado, se c um divisor comum de a e b, ento c um divisor comum de a mb e b, portanto c|d; enm, segue bem que d = (a, b).

32

Introduo Teoria dos Nmeros  [Antonio Carlos & Ana Paula Marques]

EXEMPLO 3.15 1. Verique: (2, 3) = 1; (14, 15) = 1; (4, 20) = 4; (10, 200) = 10; (10, 201) = 1
2. Vamos exemplicar o Lema de Euclides (a mb, b) = (a, b) = (a, b na) nas situaes-problemas abaixo; todas as variveis so nmeros naturais 1. a) (n, n + 1) = 1, pois (n, n + 1) = (n, (n + 1) n) = (n, 1) = 1; observe, tambm, que n e n + 1 tm paridades diferentes. b) (n, 2n + 1) = (n, (2n + 1) 2n) = (n, 1) = 1. c) (n2 + n + 1, n + 1) = (n2 + n + 1 n(n + 1), n + 1) = (1, n + 1) = 1 d) (n! + 1, (n + 1)! + 1) = (n! + 1, (n + 1)! + 1 (n + 1)(n! + 1)) = (n! + 1, n) = (n! + 1 n(n 1)!, n) =
(1, n) = 1 am 1 e) ,a 1 a1 = (m, a 1), a 2.

Inicialmente, observemos que

am 1 = am1 + am2 + + a + 1 a1 am 1 = (am1 1) + (am2 1) + + (a 1) + m = (a 1) + m a1 ((a 1) + m, a 1) = (m, a 1).

3. Vericar a relao:(a, a + b)|b. Pondo d=(a,a+b), temos d|a e d|a + b, logo d|b. 4. Vericar: se ms nr = 1, ento (ma + nb, ra + sb) = (a, b). Pondo d = (a, b), = ma + nb, = ra + sb, D = (, ), temos que, como d|a e d|b, segue d| e d| , logo d|D. Reciprocamente, temos D| e D| ; explicitando a e b das equaes ma + nb = e ra + sb = , cuja matriz inversvel, pois ms nr = 1 o valor de seu determinante, obtemos a = s n e b = m r, o que mostra que D|a e D|b, logo D|d. Como d|D e D|d, e ambos so positivos, vem d = D.

Atividade-proposta 3.16
1. Um oricultor possui 100 rosas brancas e 60 rosas vermelhas e pretende fazer o maior nmero possvel de ramalhetes iguais entre si. Quantos sero os ramalhetes e quantas rosas de cada cor deve ter cada um deles? 2. Indique os possveis valores de (n, n + 10). 3. Se (a, b) = 1, ento (a) ((a + b), (a2 ab + b2 )) = 1 ou 3; (b) encontre os posssveis valores de (a2 b2 , a3 b3 ). 4. Considere os nmeros de Fermat Fn = 2(2 ) + 1, n 0. Mostre que:
n

(a) se 0 m < n, ento Fm | (Fn 2); (b) se m = n, ento (Fm , Fn ) = 1.

3.17 Algoritmo de Euclides


Para o clculo explcito do mdc de dois inteiros dados, dispomos do mtodo de Euclides, das divises sucessivas, um bem fundamentado algoritmo, inclusive do moderno ponto de vista computacional. Sejam os inteiros a e b, que podemos logo supor a > b > 0. Usando a diviso euclidiana, temos a = bq1 + r2 , 0 r2 < b.

Divisibilidade e Algoritmo Euclidiano da Diviso

33

Pelo Lema de Euclides, vemos que (a, b) = (a bq1 , b) = (r2 , b) = (b, r2 ). Podem ocorrer dois casos: (1) r2 = 0. Nesse caso, (a, b) = (b, r2 ) = (b, 0) = b; (2) r2 = 0. Agora, prosseguimos com a diviso euclidiana, de b por r2 : b = r2 q2 + r3 , 0 r3 < r2 Com o mesmo argumento do Lema de Euclides, vem (a, b) = (b, r2 ) = (r2 , r3 ). Novamente dois casos podem se apresentar: (1) r3 = 0. Temos (a, b) = (r2 , 0) = r2 ; (2) r3 = 0. Prosseguimos com a diviso euclidiana, de r2 por r3 , e assim por diante. Obtemos uma sequncia estritamente decrescente b > r2 > r3 > r4 > > 0 de nmeros naturais, o que obriga a ocorrncia de algum rn = 0 e rn+1 = 0, tornando estacionria aquela sequncia. Por construo:
(a, b) = (b, r2 ) = = (rn , rn+1 ) = (rn , 0) = rn

onde o ltimo resto no nulo rn indica o mdc procurado. As divises sucessivas podem ser resumidas no quadro abaixo:
q1 q2 q3 a b r2 r3 r2 r3 r4 r5 qn2 qn1 qn rn2 rn1 rn rn 0

Exemplo motivador
Aliquemos o algoritmo de Euclides aos inteiros a = 4512 e b = 4128, obtendo o quadro-resumo:
1 384 288 10 96 1 0 3 4512 4128 384 288

96

Alm de indicar o mdc (4512, 4128) = 96, o algoritmo fornece uma informao surpreendente: uma combinao linear a + b = d !! A obteno dessa combinao linear feita assim: depois de indicar, por coluna, as divises sucessivas, ordenadamente, iniciando pela penltima (a que contem o resto-mdc), explicitamos cada resto e substituimos na equao anterior. Do quadro acima, temos:
384 = 1 288 + 96 = 96 = 384 1 288 4128 = 10 384 + 288 = 96 = 384 1 (4128 10 384) = 11 384 1 4128 4512 = 1 4128 + 384 = 96 = 11 (4512 1 4128) 1 4128 = 11 4512 12 4128

O processo indicado forneceu uma combinao linear (dita de Bzout): 11a 12b = d = 96.

3.18 Propriedade caracterstica


Sejam a e b inteiros no ambos nulos, e J (a, b) = {ax + by ; x, y Z} o conjunto de todas as combinaes lineares inteiras de a e b. Ento: (i) J (b, a) = J (a, b) = ; (ii) Se d > 0 menor inteiro positivo de J (a, b) (que combinao linear de a e b), ento d = (a, b) e J (a, b) = {rd ; r Z}; (iii) Em particular, existe uma combinao linear d = ax0 + by0 . As combinaes lineares a = 1 a e b = 1 b mostram que tanto a como b guram em J (a, b); na realidade, por hiptese, o conjunto J (a, b) contm inteiros no nulos. Por outro lado, se z J (a, b), ento claro que z J (a, b), logo, h inteiros positivos no conjunto considerado. Pela Boa Ordenao, existe um menor inteiro positivo d > 0 combinao linear de a e b, digamos d = ax0 + by0 .

34

Introduo Teoria dos Nmeros  [Antonio Carlos & Ana Paula Marques]

Se c|a e c|b, a combinao linear anterior mostra que c|d. Enm, mostremos que d|a e d|b. Conside-remos a diviso euclidiana de a por d: a = qd + r, 0 r < d. Segue que r = a qd = a q (m0 a + n0 b) = a(1 qm0 ) qn0 b uma combinao linear de J (a, b); ora, a condio 0 r < d e a minimalidade de d obriga r = 0, logo d|a. Analogamente, d|b. Enm, qualquer combinao linear z J (a, b) um mltiplo de d, e vice-versa.

Corolrio Sejam a, b Z e n um inteiro positivo; ento (na, nb) = n(a, b). De fato, temos J (na, nb) = nJ (a, b) = {nz ; z J (a, b)}; segue, ento, que (na, nb) = menor valor positivo de (na)x + (nb)y , onde x, y Z; como n > 0, = n { menor valor positivo de ax + by , onde x, y Z }, isto , (na, nb) = n(a, b).
Pratique um pouco!

Pondo d = (a, b) e D = (na, nb), verique que D = nd, mostrando que D|nd e nd|D.

3.19 Inteiros primos relativos


Dois inteiros a e b so primos relativos, primos entre si ou coprimos, se (a, b) = 1. Por exemplo, 10 e 21 so primos relativos, pois (10, 21) = 1. Tambm so coprimos os inteiros 2010 e 2011.

Proposio 1 Dois inteiros a e b so primos relativos se, e somente se, existem inteiros e tais que a + b = 1 (isto , alguma combinao linear de a e b igual a 1). Realmente, se (a, b) = 1, ento uma combinao de Bzout verica bem a + b = 1. Reciprocamente, partindo de uma tal combinao linear, se d = (a, b), ento d|a e d|b, logo d|1, ou d = 1.
a b , =1 d d Pondo a = da1 e b = db1 , se a + b = d, ento, simplicando por d, vem a1 + b1 = 1, isto , (a1 , b1 ) = 1.

Proposio 2 Se d = (a, b), ento

Proposio 3 Sejam a, b e c inteiros no nulos. Se a|(bc) e (a, b) = 1, ento a|c. De fato, se a + b = 1, ento, multiplicando por c, obtemos ac + bc = c; por hiptese, a|bc, ou bc = qa; segue, enm, a(c + q ) = c da forma ak = c,donde a|c. Exemplo 3.20
1. Sejam os inteiros coprimos a e b : (a, b) = 1 ; para cada inteiro c, se a|c e b|c, ento (ab)|c. Realmente, como a|c, ento existe tal que c = a; analogamente, existe tal que c = b; assim, vale c = a = b. Ora, vemos da que a|b, donde, como (a, b) = 1, segue a| ; portanto, existe k inteiro tal que = ka e c = (ka)b = k(ab), ito , (ab)|c.

Pratique um pouco! No resultado acima, a hiptese (a, b) = 1 pode ser enfraquecida?


2. Seja n 1; aps ter vericado que a = eb= so inteiros positivos, 2 6 calcule (a, b), considerando os dois casos: n = 3q ou n = 3q + 2, e n = 3q + 1. O nmero a inteiro, pois o produto n(n + 1) par [ver o exemplo 3.11(3)]; b inteiro pelo argumento de 3.9(6). Seja d = (a, b); temos
6d = (6a, 6b) = (3n(n + 1), n(n + 1)(2n + 1)) = n(n + 1)(3, 2n + 1) = n(n + 1)D, com D = (3, 2n + 1). Caso(1) n = 3q ou n = 3q + 2. Ento D = (3, 2n + 1) = 1, 6d = n(n + 1) 1 e d = n(n + 1)/6. n(n + 1) n(n + 1)(2n + 1)

Caso(2) n = 3q + 1; vemos que D = (3, 2n + 1) = 3; 6d = n(n + 1) 3 e d = n(n + 1)/2.

Divisibilidade e Algoritmo Euclidiano da Diviso


3. Se (a, b) = 1, ento (a, bc) = (a, c); concluir que (a, bc) = 1 se, e somente se, (a, b) = (a, c) = 1.

35

(a) Sejam d = (a, bc) e D = (a, c); como D|a e D|c, ento D|bc, logo D|d. Por outro lado, com (a, b) = 1, existe alguma combinao linear xa + yb = 1, donde, multiplicando por c, (xa)c +(yb)c = c; enm, como d|a e d|bc, segue que d|c, logo d|D. (b) Se (a, b) = (a, c) = 1, ento a primeira igualdade, junto com (a), nos d (a, bc) = (a, c) = 1. Reciprocamente, se (a, bc) = 1, ento uma combinao linear xa + y (bc) = 1 informa que (a, b) = 1 = (a, c).

Atividade proposta 3.21


1. Use o algoritmo de Euclides para calcular d = (a, b) e indicar uma combinao linear xa + yb = d. (a) (14, 5); (b) (33810, 4116); (c) (1000, 761) 2. Dados a = 188887 e b = 211109, calcule (a, b) e encontre os inteiros x e y tais que = , com y b x + y = 108. 3. [Continuao do exemplo 3.20, Exerccio 3] Suponhamos (a, b) = (a, d) = (c, b) = (c, d) = 1; mostre que (a) ((ac), (bd)) = 1; (b) Para todos m, n N, (am , bn ) = 1 ; (c) Em particular, se (, ) = 1, ento (n , n ) = 1. Conclua que, dados a, b Z, vale (an , bn ) = (a, b)n .
x a

3.22 Mnimo Mltiplo Comum Dados os inteiros no nulos a e b diremos que um m.m.c de a e b um inteiro m tal que (1) m mltiplo comum de a e b: a|m e b|m; (2) para todo inteiro c que seja mltiplo comum de a e de b, isto , tal que a|c e b|c, ento deve ser m|c. Tal como no caso do mdc, se o inteiro m1 tambm verica as duas condies (1) e (2), ento m|m1 e m1 |m; se xarmos da denio anterior a escolha m > 0, vemos que o mmc unicamente determinado pela condies consideradas. Usaremos, ento, a notao [a, b] para representar o mmc(a,b)> 0.
Por exemplo, [2, 3] = 6, [5, 500] = 5, [4, 10] = 20.

Proposio 1 Se a, b Z , ento |ab| = (a, b)[a, b]. Supondo logo a > 0 e b > 0, e pondo d = (a, b), a = a1 d, b = b1 d e m = [a, b], a relao ab = dm ca m = a1 b = ab1 (). Ora, essa igualdade (*) mostra que m|a e m|b. Em seguida, se a|c e b|c, ento c = xa = yb, ou xa1 = yb1 , donde b1 |xa1 , mas (a1 , b1 ) = 1, logo b1 |x e, digamos, x = qb1 , que, em c = xa, nos d c = qb1 a = qm , tendo em conta (*). Assim, m|c. Corolrio Se (a, b) = 1, ento [a, b] = |ab|. Exemplo 3.23
1. Como (10, 201) = 1, vem [10, 201] = 10 201 = 2010. (4512, 4128) = 96, logo [4512, 4128] = 4512 4128/96 = 194016. Do mesmo modo, vimos que

36

Introduo Teoria dos Nmeros  [Antonio Carlos & Ana Paula Marques]

2. Achar cada mmc, onde n N. (a) [n, 2n + 1] Como (n, 2n + 1) = 1, segue [n, 2n + 1] = n(2n + 1). (b) [n + 1, n2 + 1] Temos (n + 1, n2 + 1) = (n(n + 1) (n2 + 1)) = (n 1, n + 1) = (2, n + 1) =
1, n par 2, n impar

Logo, [n + 1, n2 + 1] = (n + 1)(n2 + 1) se n par, ou [n + 1, n2 + 1] = (n + 1)(n2 + 1)/2 se n mpar. 3. Seja M (a, b) o conjunto dos mltiplos comuns dos inteiros no nulos a e b, e seja m o menor inteiro positivo mltiplo comum de a e b, isto , m = min M (a, b) N. Para vericar que m = [a, b], basta ver que, se a|c e b|c, ento m|c. Ora, na diviso euclidiana c = qm + r, com 0 r < m, vemos que tanto c como m so mltiplos de a e b, logo, tambm r mltiplo comum deles; assim a minimalidade de m obriga r = 0, isto , m|c.

Pratique um pouco! Explique como garantir M (a, b) = ; depois, como garantir a existncia de elementos positivos em M (a, b). Atividade-proposta 3.24
1. Calcule cada mmc dos inteiros do Exerccio 1 da Atividade 3.21. 2. Numa corrida de F1, um piloto d a volta completa numa pista de treinos em 15 segundos e um outro, em 18 segundos. Mantendo-se o mesmo tempo, no nal de quantos segundos os dois pilotos passaro juntos pela primeira vez no ponto de partida e quantas voltas tero dado cada um? 3. Mostre que (a, b) = (a + b, [a, b]). 4. Dados (a, b) = 1, ache c de modo que cada um dos inteiros a, b, c divida o produto dos outros dois.

3.25 Equaes Diofantinas Lineares


Uma equao diofantina linear da forma ax + by = c (*), onde a, b e c so inteiros dados, com a e b no ambos nulos [do contrrio, todos os pontos inteiros x e y seriam soluo, se c = 0, ou a equao no teria soluo, se fosse c = 0]. Para descrever todas as solues de (*), vamos usar e abusar do mdc d = (a, b).

(1) Existncia de soluo A equao ax + by = c possui soluo se, e somente se, d = (a, b) | c. Se existe algum par de inteiros (x0 , y0 ) tal que ax0 + by0 = c, ento claro que d = (a, b)|c. Reciprocamente, se d|c, com c = qd, e se a + b = d uma combinao de Bzout, ento, multiplicando por q , vem a(q) + b(q ) = qd = c, o que mostra que x0 = q e y0 = q formam uma soluo da equao dada. Observemos o processo explcito para exibir uma particular soluo!! (2) Clculo de todas as solues J supondo que d = (a, b)|c, consideremos a chamada equao reduzida, obtida da equao dada pela diviso de todos os coecientes pelo mdc d: a1 x + b1 y = c1 (**), s que, agora, temos (a1 , b1 ) = 1 primos entre si. Formada uma soluo particular como vimos acima, temos a1 x0 + b1 y0 = c1 , que subtrada da equao (**) nos d: a1 (x x0 ) = b1 (y0 y ). Ora, b1 divisor do primeiro membro, mas primo com a1 , logo b1 |(x x0 ), isto , existe t Z tal que x x0 = b1 t, ou x = x0 + b1 t; da mesma equao, segue y0 y = a1 t, ou y = y0 a1 t. Enm, claro que levando

Divisibilidade e Algoritmo Euclidiano da Diviso


x = x0 + b1 t, y = y0 a1 t, t Z

37

na equao dada, vemos que so bem soluo. Assim, determinamos todas as solues da equao dada. Observemos que os coecientes da soluo geral so dados pela equao reduzida.

Exemplo 3.26
1. A equao diofantina 4x + 10y = 3 no possui soluo pois (4, 10) = 2 | 3. 2. A equao diofantina 4x + 10y = 6 possui soluo pois (4, 10) = 2|6. Passando para a equao reduzida: 2x + 5y = 3. Por inspeo, vemos que x0 = 4 e y0 = 1 uma soluo. Logo, todas as solues se escrevem x = x0 + b1 t = 4 + 5t, y = y0 a1 t = 1 2t, t Z. 3. Achar, se possvel, todas as solues da equao diofantina 90x 28y = 22. Usando o algoritmo de Euclides, obtemos (90, 28) = 2|22 e uma combinao linear 2 = 90 5 28 16. Da equao reduzida 45x 14y = 11 para uma soluo particular, retomamos a combinao linear tambm reduzida, isto , 45 5 14 16 = 1, atualizada para 45 55 14 176 = 11, donde x0 = 55, y0 = 176. A soluo geral vale x = 55 14t, y = 176 45t, t Z.

Atividade-proposta 3.27
1. No exerccio 2 do exemplo 3.26 acima, apresentamos a soluo geral x = 4 + 5t, y = 1 2t, t Z, da equao 4x + 10y = 6 (ou de sua reduzida equivalente 2x + 5y = 3); a soluo particular (4, 1) foi escolhida por tentativa. Usando o mtodo terico descrito, a soluo particular dada por uma combinao de Bzout, via o algoritmo de Euclides, no caso 2 = 1.10 2.4, ou 1 = 1.5 2.2 atualizada para 3.5 6.2 = 3, o que nos d a soluo (6, 3), donde a soluo geral x = 6 + 5u, y = 3 2u, u Z. O que aconteceu? Formamos uma nova famlia de solues? Verique que, fazendo a mudana de parmetros u = 2 + t, ento a segunda famlia coincide com a famlia inicial! 2. No exerccio 3 do exemplo 3.26 acima, a soluo geral x = 55 14t, y = 176 45t, t Z, pode ser reformulada para exibir, por exemplo, as solues positivas da equao 90x 28y = 22. Verique o seguinte procedimento: (1) Inicialmente, escolher o maior t para ocorrer x = 55 14t > 0 e y = 176 45t > 0; achamos t = 3, o que corresponde a x0 = 13 e y0 = 41. (2) Em seguida, mudando o sinal do parmetro, segue a soluo geral das solues positivas x = 13 + 14t, y = 41 + 45t, t N 3. Achar todas as solues inteiras da equao diofantina 30x + 17y = 201. A equao dada possui soluo positiva? 4. Resolva a equao diofantina x3 + (x + 1)3 + (x + 2)3 = (x + 3)3 . Sugesto. Faa x = t + 3.

Nmeros Primos

41

CAPTULO 4 NMEROS PRIMOS

A escola pitagrica (c. 530 AC) destacava o nmero um, a unidade (monad), cujos mltiplos geravam os demais inteiros positivos, denominados nmero (arithms). Foi o prprio Pitgoras quem sugeriu dois tipos de arithmi: os protoi arithmi (nmeros primrios ou primos), aqueles que no podem ser gerados (via multiplicao) por outros arithmi, como o caso de 2, 3, 5, 7, 11, ..., e os deuteri arithmi (nmeros secundrios), os que podem ser gerados por outros arithmi, como o caso de 4 = 2.2, 6 = 2.3, 8 = 2.4, 9 = 3.3, etc.

Objetivos do Captulo 4

(a) Estabelecer os conceitos de nmeros primos e de decomposio primria; (b) Explicitar o Teorema fundamental da Aritmtica e suas consequncias; (c) Descrever as principais funes aritmticas; (d) Retomar as propriedades do mdc e mmc luz da decomposio primria; (e) Aplicar a teoria a problemas elementares de contagem.

4.1

Introduo

Aos dezoito anos, em 1796, Gauss publicou sua construo do polgono regular de 17 lados, usando apenas os mtodos euclidianos de rgua (no graduada) e compasso. No mesmo artigo, Gauss armou que aquela construo, realmente, era um corolrio de uma teoria mais geral, que ainda estava sendo completada. Na realidade, dois anos aps, a ltima parte de sua obra prima Disquisitiones Arithmeticae dedicada ciclotomia, onde demonstra o teorema que decide a construtibilidade de um polgono regular com um nmero primo um nmero primo de Fermat.

de lados: o poligono construtvel se, e somente se,

42

Introduo Teoria dos Nmeros  [Antonio Carlos & Ana Paula Marques]

4.2

Nmeros Primos

Dado um nmero inteiro e, se

a = 1, triviais de a.

ento

a, sabemos que D(A) contm, pelos p

o conjunto

D(A)

dos divisores de

menos, quatro elementos

a no vazio (pois 1|a) a, os chamados divisores

Pode acontecer que um inteiro

p = 0, p = 1, p = 0
e

possua, apenas, os divisores triviais;

nesse caso, diremos que o inteiro

primo. Mais formalmente:

Denio Denio
d|n
tal que

Um nmero inteiro

p n

primo se: (i)

p = 1;

(ii) os nicos divisores de

so

1, 1, p, p.
Um nmero inteiro composto se

n = 0, n = 1,

e existe algum divisor (no trivial)

1 < |d| < |n|. p > 1 primo se os nicos divisores forma n = ab, com 1 < a, b < n. p>2
positivos de

Em particular, um inteiro

so

p;

um inteiro

n>1

composto se da

Por exemplo, so primos os inteiros 2,3,5,7,11,13,31; os inteiros 6,8,9,10,12 so compostos. inteiro par

Todo

n>2

composto. Todo inteiro primo

mpar.

4.3 Propriedades

(1) Seja p Z um inteiro primo.


(i) dado (ii) se

Temos:

a Z,

se

p |a,

ento

(p, a) = 1;
ou

a, b Z p|ab,

ento

p|a

p|b.
logo, como

O item (i) decorre do seguinte fato: se

d = 1;

ora, no pode ser

d = p,

pois

(p, a) = d, ento d|p e d|a, p |a. Enm, d = 1 = (p, a). (p, a) = 1.


Logo, Seja

primo,

d=p

ou

Quanto a (ii), se

p|ab

p |a,

ento, como vimos,

p|b. |a| 2;
ento,

(2)  Existncia de nmeros primos


divisor primo. Se

um inteiro tal que

possui algum

primo, no h nada a vericar. Suponhamos

a2

um inteiro composto positivo. Ento,o Do

S = {m ; m|a e 1 < m < a} no vazio. Armamos que d = min S primo. contrrio, se b|d e 1 < b < d, ento b|a e 1 < b < d < a, contradizendo a minimalidade de d.
conjunto

Corolrio 1 Se p 2 o menor divisor positivo de um inteiro a 2, ento p primo. Corolrio 2 Se a 2 um inteiro composto, ento a possui algum divisor primo positivo p tal que
p 2 a.
De fato, se

a = pq
Se

o menor divisor (primo) de Seja

a,

ento

q p,

donde

pq p2 ,

ou

a p2 .

(4)  Critrio elementar de primaridade


2 que d

a2

um inteiro e

o maior inteiro positivo tal

< a.

[2, d] D(a) = ,

ento

um inteiro primo.

a = 491, indicamos de a pelos primos= 2, 3, 5, 7, 11, . . .. 232 > 491; logo, a = 491 primo!
Por exemplo, com

na tabela abaixo, os respectivos quocientes e restos da diviso Ora, nenhuma diviso foi exata at

p = 23

e, na realidade,

p
2 3 . . . 19 23

q
245 163

r
1 2

p2
4 9 . . .

25 21

16 8

361 529

Nmeros Primos

43

A seguir, vericaremos que h um estoque sucientemente grande de nmeros primos!

4.4 O conjunto dos nmeros primos {2, 3, 5, 7, } innito.


A vericao remonta a Euclides. Suponhamos, por absurdo, que a lista nita todos os primos. Seja o inteiro primo

N = p1 p2 pm + 1;

como todo inteiro

{p1 , p2 , . . . , pm } esgota 2, N possui algum fator


pois se isso fosse vlido,

p;

esse fator primo no pode ser nenhum dos primos

p1 , p2 , . . . , pm ,

ento, obrigatoriamente, seria

p|1,

o que manifestamente no ocorre! Portanto, o esquema anterior

aponta para um nmero primo distinto de qualquer lista nita de primos. Por exemplo, partindo de anteriores; usando

p1 = 2 e p2 = 3, obtemos p1 p2 + 1 = 7, um primo p3 = 5 e p4 = 7, obtemos p3 p4 + 1 = 36 que no primo, mas

distinto dos dois divisvel por

2,

primo distinto dos dois anteriores.

4.5 Teorema Fundamental da Aritmtica


Todo nmero inteiro

a = 0 e a = 1 igual a um produto de nmeros primos; a = p1 p2 pr = q1 q2 qs so duas tais decomposies, ento r = s e, a menos fatores, pi = qi , para i = 1, 2, . . . , r .
De fato, podemos supor que

alm disso, se da ordem dos

a > 1. Seja S o conjunto dos inteiros a 2 que so produtos de primos; observemos que 2 S . Seja a > 2 e suponhamos o resultado vlido para cada inteiro r tal que 2 r < a. Se a primo, ento a S . Se a no primo, ento existem inteiros b e c tais que a = bc, com 2 b < a e 2 c < a. Assim b S e c S , ou seja b e c so produtos de nmeros primos, logo a = bc tambm um produto de primos, isto , a S . Em virtude do segundo princpio da induo nita, todo inteiro a 2 um produto de nmeros primos. a
positivo, logo, Quanto unicidade, suponhamos

p1 p2 pr = q1 q2 qs ; como p1 |q1 q2 qs , ento p1 = qj para algum j , que podemos supor que seja q1 , aps um reordenamento conveniente de q1 q2 qs . Portanto, p2 pr = q2 qs , logo, pela hiptese de induo, r = s e os pi e qj so iguais aos pares.

4.6  Decomposio Primria


No resultado anterior, podemos agrupar os eventuais fatores primos repetidos; ordenando os fatores em ordem crescente, obtemos o seguinte enunciado. Dado um inteiro positivos

a Z , |a| = 1, existem primos positivos p1 < p2 < < pm 1 , 2 , , m , unicamente determinados, tais que
2 m 1 a = p 1 p2 pm
se

e nmeros inteiros

onde

=1

a > 0,

= 1

se

a < 0.
2 1 m a = p p m 1 p2

Por exemplo,

32 = 25 ; 12 = 22 3; 1000 = 23 53 ; 10890 = 2 32 5 112 .


um quadrado perfeito se, e

Pratique um pouco Um nmero natural


somente se, cada expoente

par.

Conveno Quando consideramos as fatoraes primas de dois ou mais nmeros, podemos supor que
comparecem os mesmos fatores primos, desde que os respectivos expoentes possam ser considerados como nmeros positivos ou nulos; um expoente nulo

p0 = 1
1 p 1

de um primo, na realidade, signica a

ausncia desse primo! Assim, por exemplo, dados os inteiros

a, b 2,

podemos escrever

a=
usando os mesmos primos Por exemplo, os nmeros

k 1 p 1 pk

e e

b=

k p k

p1 , . . . pk ,

onde

j 0

j 0, 1 i, j k . 24 32 50 7 11 e 2 30 52 7 110 .

24 32 7 11 e 2 52 7,

podem ser escritos

44

Introduo Teoria dos Nmeros  [Antonio Carlos & Ana Paula Marques]

Mantida a conveno anterior, sejam

k 1 a = p 1 pk

k 1 b = p 1 pk ,

com

j 0

j 0,

1 i, j k .

Valem as relaes.

4.7 Propriedades
(a)

a|b j j ; dj = min{j , j },
ento ento

(b) Pondo (c) Com

dk 1 d = (a, b) = pd 1 pk ; mk 1 m = [a, b] = pm 1 pk .
Supondo que

mj = max{j , j },

Na realidade, basta vericar o item (a).

a|b,

se

a potncia de um primo
i p i , donde

gura na decomposio de a, ento p |b, logo


recproca evidente.

p divide algum

p = pi

p que i . A

Pratique um pouco Verique os detalhes evidentes dos resultados acima.

EXEMPLO 4.8 1.
Usando o critrio

4.3(4), decidir se os nmeros 373, 667 e 4.001 so primos.


373 = 19 19 + 12,
vemos que

O nmero

373

no divisvel por 2,3,5,7,11,13,17,19; como

373

primo. Do mesmo modo, primo o nmero

4001 = 67 59 + 48.

O inteiro

4.001, pois no 667 = 23 29 composto.

divisvel por 2,3,5,7,11,13,17,19,...,67 e

2.

Achar a decomposio primria, o mdc e o mmc dos inteiros e

b.

a = 10.890

b = 6.825
2 3 3 5 11 11 6825 2275 455 91 13 1 3 5 5 7 13

10890 5445 1815 605 121 11 1

Vemos,ento, que

a = 2 32 5 112 e b = 3 52 7 13, donde d = (a, b) = 3 5 = 15 e m = [a, b] = 2 32 52 7 112 13 = 4.954.950.


Sejam

3.

a, b N , ab =

com

(a, b) = 1. n > 0,

Se

ab

um quadrado, ento

so quadrados.

Aplicao.Para cada inteiro De fato, se

o produto

n(n + 1)

no um quadrado. em

m2 , ento cada fator primo de

mesmo fator gura tambm no primeiro

m gura membro ab, mas

m2

com um expoente par. Logo, esse

somente em um dos nmeros

ou

b,

pois

a e b contm apenas fatores primos com expoente par, isto , so quadrados. Para a aplicao numrica, como (n, n + 1) = 1, se fosse n(n + 1) um quadrado, o mesmo seria para n e n + 1, digamos n = a2 e n + 1 = b2 , ou (n + 1) n = 1 = b2 a2 , donde b + a = 1 e b a = 1, ou b + a = 1 e b a = 1; em qualquer caso, seria n = 0, contra a hiptese. p p 4. Para cada nmero binomial , onde p 2 primo e 1 k p 1, vale p . k k
Enm, ambos Como

(a, b) = 1.

k!

p k

divisor de

= p(p 1) (p k + 1), claro p k !, isto , (p, k !) = 1, donde p| k . 5

que

p|k !

p k ; ora, sendo

1 k p 1, p n>0

no , ento,

5.
e

Veriquemos que

o nico primo da forma

n4 + 4 ,
ento

para algum inteiro ou

Ora, se

5 = n4 + 4 = (n2 + 2n + 2)(n2 2n + 2), n2 + 2n + 2 = n4 + 4 = 5.

n2 2n + 2 = 1,

(n 1)2 = 0,

donde

n=1

Nmeros Primos 6.
O exemplo a seguir ilustra a irregularidade da distribuio dos primos em

45

N.

(a) Dado

n 1, observe que a sequncia m, m + 1, . . . , m + n 1, com m = (n + 1)! + 2,

formada por

inteiros consecutivos compostos!

E tem mais! No somente exibimos uma sequncia formada de inteiros consecutivos e compostos, de tamanho arbitrrio, como tambm exibiremos uma innidade de tais sequncias!! Para cada inteiro

t 1,

a sequncia

M, M + 1, . . . , M + n 1,
formada por

com

M = t(n + 1)! + 2

inteiros consecutivos compostos!

Atividade-proposta 4.9
1. Usando o critrio

4.3(4), decidir se os nmeros 171, 761, 3.881 so primos.


(2)

2. Achar a decomposio primria, o mdc e o mmc dos pares de inteiros: (1)

a = 1.000 n2

b = 761;

c = 10.890

6.825. fn
tambm composto.

3. Se

um inteiro composto ento o termo de Fibonacci

Sugesto.Rever o exerccio

3.5(6), onde vimos que m|n fm |fn .


seja primo, a escolha de

Pratique um pouco Pelo resultado acima, para que fn


4. Calcule todos os valores do inteiro Sugesto. Se

nmero primo. Ser que basta uma tal escolha?! Verique que

f53

n deve ser a de um = 53.316.291.173 no primo.


um quadrado.

n0

para os quais

n2 + n + 43
ou

n2 + n + 43 = a2 p2

, ento

4n2 + 4n + 172 = 4a2 , p

(2n + 1)2 + 171 = 4a2 . (p 1)! + 1,


ento

5. [Wilson] Seja

um inteiro positivo. Se

um divisor de

primo.

Sn = 1! + 2! + n!. Encontre n para que a soma Sn seja um Sugesto. Temos S1 = 1, S2 = 1 + 2 = 3, S3 = 1 + 2 + 6 = 9, S4 = 33; agora Sn n 5, pois, nesse caso, n! multiplo de 10.
6. Considere a soma

quadrado. termina por 3 se

4.10 Aplicaes da decomposio em fatores primos


k 1 a 2, a = p 1 pk , com j 0, 1 j k . dk d1 Como vimos na propriedade 4.3(a), os divisores d|a possuem decomposio da forma d = p1 pk , j onde 0 dj j . Observemos que cada primo pj de a contribui com (j + 1) divisores, a saber j {1, pj , p2 j , . . . , pj }. Como essas contribuies so independentes dos primos considerados, teremos um total (a) de divisores, dado por:
Consideremos a decomposio primria do inteiro

(a) = (1 + 1)(2 + 1) (k + 1)
Na realidade, esses divisores so todos os produtos da forma j considerada acima

dk 1 pd 1 pk ,

com

0 dj j .
Por exemplo, se

3 2 = 6,

p primo, ento (p) = 2; (8) = (23 ) = 4, 2 saber {1, 2, 3, 4, 6, 12}; (525) = (3 5 7) = 12

a saber

{1, 2, 4, 8}; (12) = (22 3) =

Observemos que a obteno da fatorao prima tambm permite encontrar os divisores. Por exemplo:

525 175 35 7 1

3 5 5 7

1 3 5-15 25-75 7-21-35-105-175-525

46

Introduo Teoria dos Nmeros  [Antonio Carlos & Ana Paula Marques]

2. Soma dos divisores


Retomando a decomposio primria do inteiro usando a propriedade

k 1 a 2, a = p 1 pk ,

com

j 0, 1 j k ,

4.3(a),

temos a igualdade

k 1 (1 + p1 + + p 1 ) (1 + pk + + pk ) =
onde a soma do membro direito tomado sobre todas as k-uplas a condio

dk 1 pd 1 pk
onde cada

(d1 , . . . , dk ),

dj

varia sob

0 dj j ;

tal somatrio representa, ento, a soma de todos os divisores de

a.

Por

outro lado, no primeiro membro, cada soma pode ser calculada pela frmula da soma de uma PG. Em denitivo, representando por

(a)

a soma dos divisores positivos de

a,

temos:

k +1 1 +1 p 1 p 1 1 k (a) = p1 1 pk + 1

Corolrio A funo (n) multiplicativa, isto , se (a, b) = 1, ento (a b) = (a) (b).


Por exemplo, se

p2

primo, ento

(p) =

p2 1 = p + 1, p1

como esperado!

3. Produto dos divisores


k 1 a = p 1 pk , com j 0, 1 j k , verique a seguinte relao, do produto P (a) dos divisores de a, onde (a) indica o nmero dos divisores positivos de a: P (a) = a(a)

Pratique um pouco! Sempre com as notaes

EXEMPLO 4.11 1.
(a) Achar todos os inteiros positivos

que vericam as relaes (b)

a2 = b2 + 1517

a + b = 5.664 (a, b) = 354

(a + b)(a b) = 1.517 ; os divisores de 1.517 sendo 1, 37, 41 e 1517, segue que a + b = 1.517 e a b = 1, ou a + b = 41 e a b = 37, donde a = 759 e b = 758, ou a = 39
(a) Temos (b) Pondo

b = 2;

a = 354a1 e b = 354b1 , o sistema dado equivale a a1 + b1 = 16 e (a1 , b1 ) = 1 . {a1 , b1 } = {1, 15}, {3, 13}, {5, 11}, {7, 9} , donde, {a, b} = {354, 5.310}, {1.062, 4.602}, {1.770, 3.894}, {2.478, 3.186} .

Segue, ento:

a e sua soma (a). 2 1 5 1 3 3 = 15 156 = 2340. Temos 1000 = 2 5 , donde (a) = 4 4 = 16 e (a) = 21 51 Pratique um pouco! Use um mtodo sistemtico para listar todos os divisores de a = 1000.
Seja calcular o total dos divisores positivos de

2.

a = 1.000;

(a)

3.

Achar os inteiros positivos

tais que

(a, b) = 1.250,

sabendo que

possui 21 divisores e

possui 10 divisores.

(a, b) = 1.250 = 2 54 , segue que a = 21 52 , b = 21 52 , com 1 , 1 1 e 2 , 2 4, donde (1 + 1)(2 + 1) = 21 = 3 7 e (1 + 1)(2 + 1) = 10 = 2 5; assim, 1 = 2, 2 = 6, 1 = 1 e 2 = 4. 2 6 4 Enm, a = 2 5 e b = 2 5 .
Como

4.

k 1 a 2, com a decomposio primria a = p 1 pk possui uma quantidade mpar (a) = (1 + 1) (k + 1) de divisores, ento cada j par, ou seja, a um quadrado perfeito.
Se o inteiro

Nmeros Primos
Aplicao.

47

Numa escola h um corredor com 1000 armrios numerados de 1 a 1000, inicialmente Mil alunos, numerados de 1 a 1000, passam pelo corredor. O aluno de nmero

todos fechados.

reverte o estado de todos os armrios cujos nmeros so mltiplos de

 por exemplo, o aluno de

nmero 5 mexe nos armrios de nmeros 5,10,15,20,..., abrindo os que encontra fechados e fechando os que encontra abertos. abertos? Ao nal, depois da passagem do milsimo aluno, quais armrios caro

Pratique um pouco!
somente se,

verique a armao:

a porta do

m-simo

armrio estar aberta se, e

for um quadrado perfeito!

5.

Determine com quantos zeros consecutivos termina a representao decimal nmero natural 1000!

Basta observar que cada zero est associado presena de um fator 2 e de um fator 5 em 1000! Levando em conta que existem mais fatores 2 que fatores 5 em 1000!, suciente achar o expoente correto de 5 na fatorao considerada. Ora, em 8 de

{1, 2, 3, . . . , 1000} h 200 mltiplos de 5, 40 de 25 = 52 ,

125 = 53

e 1 de

625 = 54 ,

num total de 249. Assim, 1000! termina em 249 zeros.

6.

Pratique um pouco Imitando a demonstrao de Euclides sobre a innitude dos primos,

vericar que h uma innidade de primos da forma Sugesto. Se primo do

4n + 3,

com

inteiro.

p0 = 3, p1 , p2 , . . . , pk so todos os primos da forma considerada, examine um fator inteiro N = 4p1 p2 pk + 3 e observe que todo primo = 2 da forma 4n + 1 ou 4n + 3. a>0
e

[O resultado caso particular de um famoso teorema de Dirichlet: se si, ento existe uma innidade de primos da forma

b>0

so primos entre Na realidade, a

a + bn (em progresso aritmtica).

construo de Dirichlet deu incio chamada Teoria Analtica dos nmeros, ao incorporar ferramentas de anlise real e complexa aos mtodos discretos clssicos. Uma famosa conjectura, sobre os zeros no reais da funa

de Riemann, teria sido formulada nesse novo quadro terico.]

Atividade-proposta 4.12
1. Ache todos os inteiros positivos (a)

vericando cada grupo de condies. ; (b)

ab = 51.840 [a, b] = 2.160

(a, b) = 5 [a, b] = 8.160 a;

2.

Dado o inteiro

de divisores de positivos de

a = 360, a em Z; (c)

calcule: (a) o nmero

(a)

dos divisores positivos de (d) a soma

a listagem dos divisores positivos de

a; (b) o nmero (a) dos divisores

a;

(e) a soma dos divisores de

em

Z.

3. Encontre o menor nmero positivo que possui 9 divisores. 4. Ache todos os inteiros positivos

tais que

[b, 12] = 60.


com

5. Considere a decomposio primria 6.Dado o inteiro a soma dos

a = p q ,

(a) = 6

(a) = 28;

calcule o inteiro onde

a.
denota

a = 210 35 5 72 13 19 23 89, divisores positivos de a. x, y Z


tais que

mostre que

(a) = 5a,

(a)

7. Achar todos os inteiros

(x 6)(y 6) = 18.

Aplicao Encontre todos os tringulos retngulos cujos lados tm medidas expressas por nmeros inteiros e tais que o dobro da rea numericamente igual ao triplo do permetro.

48

Introduo Teoria dos Nmeros  [Antonio Carlos & Ana Paula Marques]

4.13 Nmeros primos de Fermat e de Mersenne


Passaremos a descrever as propriedades de duas formas de nmeros primos, que continuam a exibir resultados fundamentais em um vasto leque de aplicaes. A primeira forma corresponde aos chamados nmeros de Fermat, mencionados por Pierre de Fermat (1601-1665), considerado, aps Euclides e Erasttenes, como o primeiro matemtico a contribuir para o desenvolvimento terico da Teoria dos Nmeros. Proposio 1 com Se Sejam

a>1

n>1

nmeros naturais. Se

an + 1

primo, ento

par e

n = 2m ,

m N.

a no fosse par, ento an + 1 seria par e maior do que dois, contra o fato de ser primo. Tambm, se n tivesse um divisor primo p = 2, ento seria n = pq , com q N , e aq + 1 dividiria (aq )p + 1 = an + 1 n m [rever 3.4], contra a hiptese a + 1 primo. Enm, n bem da forma 2 .
Os

nmeros de Fermat so os nmeros da forma Fn = 22n + 1.


F1 = 5, F2 = 17 , F3 = 257
so os primeiros nmeros primos de Fermat. No se sabe se existem outros primos de

Fermat acreditava que todos esses nmeros eram primos. Na realidade, e

F4 = 65537

Fermat alm dos quatro primeiros. Vimos anteriormente [3.164(b)] que, se

n = m,

ento

(Fn , Fm ) = 1.

Pratique um pouco!
nmeros primos. Proposio 2 Se fosse Logo, Se Sejam ento

Use o resultado acima para uma outra prova de que existem innitos

a > 2, a = 2.

a > 1 e n > 1 nmeros naturais. Se an 1 primo, ento a = 2 e n a 1 > 1; como a 1|an 1 (ver prop. 3.4), ento an 1 no seria n
da forma

primo. primo.

n no primo, r (2 )s 1 = 2n 1,
Os

ento

n = rs,

que, ento, no seria primo!

r > 1 e s > 1, Logo, n primo.


com

donde seria

2r 1

divisor de

nmeros de Mersenne so da forma Mp = 2p 1, onde p um nmero primo.


Mp
que correspondem aos nmeros primos A relao completa dos primos de Mersenne conhecidos pode ser consultada Nesse endereo, vemos o maior nmero primo conhecido atual-

Por exemplo, so primos de Mersenne os nmeros

p = 2, 3, 5, 7, 13, 19, 31.

em http://primes.utm.edu/.

mente (de 2008); o nmero de Mersenne grande,n?).

243112609 1,

que possui 12978189 dgitos decimais (meio

O interesse, hoje em dia, em bem estudar esses nmeros, decorre dos gigantescos nmeros primos que podem representar; como os

Mp

so formados de potncias de 2, h algoritmos computacionais

que decidem sua primaridade num tempo de operao razovel. Essas caractersticas favorecem seu emprego em aplicaes do tipo criptograa inteira, que comentaremos mais tarde [nal do captulo 5].

Exemplo 4.14
1. Observe o correto resultado da proposio 2: se

2n 1

primo, ento

primo; nada se arma

sobre a recproca, o que explica as lacunas nos nmeros de Mersenne no um nmero primo. 2. Se

Mp .

Assim,

M11 = 2.047 = 23 89

so primos distintos, ento

(Mp , Mq ) = 1,

onde

Mp = 2p 1

Mq = 2q 1.
pois

Basta usar o resultado:

(Mp , Mq ) = (2p 1, 2q 1) = 2(p,q) 1 = 2 1 = 1,

(p, q ) = 1.

Nmeros Primos

49

4.15 Nmeros Perfeitos


Um inteiro

n>0

um

nmero perfeito se (n) = 2n (onde denota a soma dos divisores positivos).


23 1 8 = 7 8 = 56. 21 n = 496 = 24 31 e n = 8128 = 26 127.

Por exemplo, so perfeitos os inteiros:

n = 6,

pois

(6) = (2 3) = (2) (3) = 3 4 = 12; n = 4 7 = 22 7,


vemos que

n = 28,

pois de

(28) =

Pratique um pouco!

So perfeitos os inteiros

Para quem aprecia leis de formao muito regulares, examinar a escrita binria dos quatro nmeros perfeitos acima: 110, 11100, 111110000, 1111111000000. Todos os nmeros perfeitos conhecidos atualmente so

pares;

no se sabe nada sobre os nmeros

perfeitos mpares. Na realidade, temos o seguinte resultado.

Teorema (Euclides-Euler) Um nmero natural n um nmero perfeito par se, e somente se, n da
forma Se

n = 2p1 Mp ,

onde

Mp

um primo de Mersenne.

(2p1 , 2p 1) = 1; p p1 M ) = (2p1 ) (M ) = 2 1 2p = 2n. usando a propriedade multiplicativa de , vem (n) = (2 p p 21 Portanto, n perfeito. p1 a maior potncia de 2 que divide n. Ento, p > 1 Reciprocamente, seja n perfeito e par. Seja 2 p1 b, com b mpar. Logo, (2p1 , b) = 1 e (n) = (2p 1) (b); como (n) = 2n, vem e n = 2 (2p 1) (b) = 2p b (*). Como (2p 1, 2p ) = 1, vem que 2p 1|b, donde existe c tal que b = c(2p 1), p p com c < b; tambm, b + c = 2 c. Levando em conta (*), temos (b) = 2 c = b + c. Enm, segue que c e b so os nicos divisores positivos de b, com b primo e c = 1. Logo, n = 2p1 (2p 1), com 2p 1 n
da forma dada, ento

p > 1,

logo

par. Como

2p 1

mpar, temos que

primo.

Atividade-proposta 4.16
1. Mostre que so perfeitos os nmeros abaixo, usando a denio ( (n) exibindo a forma

= 2 n)

e, alternativamente,

n=

2p1 M

p do Teorema de Euclides-Euler.

(a) 496; (b) 8128; (c) 33550336 2. Dado o nmero perfeito

n = 28,

faa a lista dos divisores positivos de

n;

em seguida, mostre que

a soma dos inversos desses divisores vale 2, isto ,

d|28

1 = 2. d

Em geral, mostre que a soma dos inversos dos divisores de um nmero perfeito igual a 2. 3. Partindo, novamente, do nmero perfeito para achar o produtos dos divisores:

n = 28 = 22 (23 1), d = 21952 = 283 .

use a lista dos divisores positivos

d|28
Generalize: o produto dos divisores do nmero perfeito 4. Se

n = 2p1 Mp

igual a

np .

um nmero perfeito, ento

8n + 1

um quadrado perfeito.

50

Introduo Teoria dos Nmeros  [Antonio Carlos & Ana Paula Marques]

4.17
Tabela dos Nmeros Primos positivos

<

4000

2 59 137 227 313 419 509 617 727 829 947 1051 1171 1289 1427 1523 1621 1753 1879 2011 2131 2269 2381 2521 2659 2749 2879 3019 3169 3307 3433 3547 3673 3803 3929

3 61 139 229 317 421 521 619 733 839 953 1061 1181 1291 1429 1531 1627 1759 1889 2017 2137 2273 2383 2531 2663 2753 2887 3023 3181 3313 3449 3557 3677 3821 3931

5 67 149 233 331 431 523 631 739 853 967 1063 1187 1297 1433 1543 1637 1777 1901 2027 2141 2281 2389 2539 2671 2767 2897 3037 3187 3319 3457 3559 3691 3823 3943

7 71 151 239 337 433 541 641 743 857 971 1069 1193 1301 1439 1549 1657 1783 1907 2029 2143 2287 2393 2543 2677 2777 2903 3041 3191 3323 3461 3571 3697 3833 3947

11 73 157 241 347 439 547 643 751 859 977 1087 1201 1303 1447 1553 1663 1787 1913 2039 2153 2293 2399 2549 2683 2789 2909 3049 3203 3329 3463 3581 3701 3847 3967

13 79 163 251 349 443 557 647 757 863 983 1091 1213 1307 1451 1559 1667 1789 1931 2053 2161 2297 2411 2551 2687 2791 2917 3061 3209 3331 3467 3583 3709 3851 3989

17 83 167 257 353 449 563 653 761 877 991 1093 1217 1319 1453 1567 1669 1801 1933 2063 2179 2309 2417 2557 2689 2797 2927 3067 3217 3343 3469 3593 3719 3853

19 89 173 263 359 457 569 659 769 881 997 1097 1223 1321 1459 1571 1693 1811 1949 2069 2203 2311 2423 2579 2693 2801 2939 3079 3221 3347 3491 3607 3727 3863

23 97 179 269 367 461 571 661 773 883 1009 1103 1229 1327 1471 1579 1697 1823 1951 2081 2207 2333 2437 2591 2699 2803 2953 3083 3229 3359 3499 3613 3733 3877

29 101 181 271 373 463 577 673 787 887 1013 1109 1231 1361 1481 1583 1699 1831 1973 2083 2213 2339 2441 2593 2707 2819 2957 3089 3251 3361 3511 3617 3739 3881

31 103 191 277 379 467 587 677 797 907 1019 1117 1237 1367 1483 1597 1709 1847 1979 2087 2221 2341 2447 2609 2711 2833 2963 3109 3253 3371 3517 3623 3761 3889

37 107 193 281 383 479 593 683 809 911 1021 1123 1249 1373 1487 1601 1721 1861 1987 2089 2237 2347 2459 2617 2713 2837 2969 3119 3257 3373 3527 3631 3767 3907

41 109 197 283 389 487 599 691 811 919 1031 1129 1259 1381 1489 1607 1723 1867 1993 2099 2239 2351 2467 2621 2719 2843 2971 3121 3259 3389 3529 3637 3769 3911

43 113 199 293 397 491 601 701 821 929 1033 1151 1277 1399 1493 1609 1733 1871 1997 2111 2243 2357 2473 2633 2729 2851 2999 3137 3271 3391 3533 3643 3779 3917

47 127 211 307 401 499 607 709 823 937 1039 1153 1279 1409 1499 1613 1741 1873 1999 2113 2251 2371 2477 2647 2731 2857 3001 3163 3299 3407 3539 3659 3793 3919

53 131 223 311 409 503 613 719 827 941 1049 1163 1283 1423 1511 1619 1747 1877 2003 2129 2267 2377 2503 2657 2741 2861 3011 3167 3301 3413 3541 3671 3797 3923

Pratique um pouco! Complete os nmeros primos ausentes da ltima linha acima.

Aritmtica Modular

53

CAPTULO 5 ARITMTICA MODULAR


Euler (Basileia, 15 de abril de 1707  So Petersburgo, 18 de setembro de 1783),

Leonhard Paul

lho de um modesto pastor protestante, foi um matemtico e fsico suo que passou a maior parte de sua vida na Rssia e na Alemanha. Dotado de extraordinria memria, e de uma habilidade invulgar para o clculo numrico efetivo, escreveu sobre os mais variados assuntos como a teoria das funes, clculo diferencial e integral, nmeros complexos, acstica, msica, teoria dos nmeros, teoria das parties, mecnica e astronomia. Quando descansava um pouco, aproveitava para criar novas teorias, como foi o caso dos grafos (eulerianos), ao resolver o problema das sete pontes de Knigsberg. Sua volumosa produo cientca, durante muito tempo, ocupou a metade de cada volume dos Anais da Academia de So Petersburgo; ainda aps 48 anos de sua morte, trabalhos seus continuaram a ser publicados nos Anais !

Objetivos do Captulo 5

(a) Estabelecer as propriedades aritmticas do anel dos inteiros modulares; (b) Explicitar a primaridade dos inteiros nas questes de inversibilidade modular; (c) Descrever o Pequeno Teorema de Fermat e sua generalizao via o indicador de Euler; (d) Estabelecer as propriedades dos nmeros de Mersenne e dos nmeros perfeitos; (e) Estudar as congruncias lineares e o Teorema Chins do Resto; (f) Descrever algoritmos de criptograa inteira (RSA).
5.1 Introduo

A civilizao chinesa parece que foi a primeira cultura em estar interessada na aritmtica modular. Realmente, entre 280 DC e 480 DC, o Manual de aritmtica do mestre Sun j indicava o que, no sculo XVII, passou a ser conhecido como o Teorema Chins do Resto. Anteriormente, matemticos chineses (400 AC) teriam estudado o caso particular 2p 2 (mod p) do Pequeno Teorema de Fermat (1636), notvel resultado que estabelece ap a (mod p). Na realidade, as formulaes denitivas desses e muitos outros resultados anunciados por Fermat foram corretamente provados, e muitos, generalizados por Euler(1736) e Gauss (1801). Em sua famosa Disquisitiones Arithmeticae, Gauss estabeleceu as propriedades das congruncias inteiras e explicitou inmeras aplicaes, dentre as quais os resultados sobre Reciprocidade Quadrtica.

54

Introduo Teoria dos Nmeros  [Antonio Carlos & Ana Paula Marques]

5.2

Congruncias inteiras

Dado o inteiro no nulo m, dois inteiros ae b so congruentes mdulo m se os restos das divises de a e b por m forem iguais. Nesse caso, usaremos a notao a b mod m. Por exemplo, 12 0 mod 3, 12 2 mod 5, 19 1 mod 4. Na relao acima, observemos que podemos sempre supor m 2; antes de mais nada, o caso m = 1 no apresenta maior interesse, j que a b mod 1 quaisquer que sejam a, b. Por outro lado, fcil ver que a b mod m a b mod (m). Uma forma alternativa de vericar a congruncia dada abaixo.

Proposio 1 a b mod m m|(a b)


Se a b mod m, ento existem inteiros r, q, q1 , tais que a = q m + r e b = q1 m + r, logo a b = (q q1 )m, donde m|(a b). Reciprocamente, suponhamos que m|(a b); pela diviso euclidiana por m, temos a = q m + r, 0 r < m e b = q1 m + r1 , 0 r1 < m, logo a b = (q q1 )m + r r1 . Segue da hiptese que m|(r r1 ), donde, como |r r1 | < m, vem r = r1 e a b mod m.

Proposio 2 A congruncia uma relao de equivalncia em Z.


De fato, valem as propriedades: (a) [reexiva] a a mod m; (b) [simtrica] a b mod m = b a mod m; (c) [transitiva] a b mod m e b c mod m = a c mod m. As propriedades (a) e (b) sendo imediatas, veriquemos (c). Por hiptese, m|(a b) e m|(b c), logo m|(a b) + (b c), isto , m|(a c), donde a c mod m.

Proposio 3 A congruncia compatvel com as operaes de soma e produto em Z, isto , valem


as propriedades: (d) Se a b mod m e c d mod m, ento a + c b + d mod m; (e) Se a b mod m e c d mod m, ento a c b d mod m; Se m|(a b) e m|(c d), ento vemos que (a b) + (c d) = (a + c) (b + d) = m|[(a + c) (b + d)], ou a + c b + d mod m; (ac bd) = a(c d) + d(a b) = m|(ac bd), ou a c b d mod m. (f) Em particular, a b mod m e n 1, ento an bn mod m. O resultado segue de (e), por induo sobre n.
Exemplos 5.3

(1) Em cada caso, dados a e m, achar o resto da diviso de a por m.


(a) a = 436543 ; m = 8 Observando que 4365 5 mod 8, temos 43652 25 1 mod 8; como 43 = 42 + 1 = 2 21 + 1, segue que 436543 = (43652 )21 4365 4565 5 mod 8, donde 5 o resto procurado. (b) a = 61987 ; m = 37 Temos: 62 = 36 1 mod 37, logo 61987 = 6 61986 = 6 (62 )993 6 31 mod 37, donde o resto vale 31. (c) a = 710 ; m = 51 Como 72 = 49 2 mod 51, segue 710 = (72 )5 (2)5 = 32 19 mod 51. Logo, o resto procurado 19.

Aritmtica Modular (2) Veriquemos que 45|133n + 173n para cada natural mpar n.
Temos: 133 = 132 13 34 13 = 442 37 8 mod 45; analogamente: 173 = 172 17 19 17 = 323 8 mod 45; enm, como n mpar, segue 133n + 173n (8)n + 8n 0 mod 45.

55

(3) Mostrar que no existe inteiro n 1 para o qual 2n + 1 divisvel por 7.


Observemos que 21 2, 22 4, 23 1 mod 7, e que esse padro de resultados se repete ciclicamente. No h, portanto, potncia de 2 congruente a 1 6 mod 7.
111

(4) Achar o resto da diviso de


i=1

i! por 12.

Observemos que, para i 4, temos 12|i!, logo, mod12, ca apenas 1! + 2! + 3! = 1 + 2 + 6 = 9.

(5) Vericar que o nmero 22225555 + 55552222 um mltiplo de 7.


Temos: 2222 3 mod 7 e 36 1 mod 7; como 5555 = 5 + 6 925, vem 22225555 35 5 mod 7. Por outro lado, de 5555 4 mod 7, 43 1 mod 7 e 2222 = 2+3 240, vemos que 55552222 42 2 mod 7. Enm, 22225555 + 55552222 5 + 2 0 mod 7.

(6) Achar o algarismo das unidades de 3100 .


Trata-se de encontrar o resto na diviso por 10. Temos 32 = 9 1 mod 10, donde 3100 = (32 )50 1 mod 10. Logo, vale 1 o algarismo procurado.

Atividade-proposta 5.4
1. Em cada caso, encontre o resto da diviso por m. (a) 67862011 , m = 11; (b) 12233 455679 876532 , m = 4 (c) a = (116 + 1717 )21 ; m = 8 2. Para cada nmero natural n, mostre que 7 divisor de 32n+1 + 22n+2 . 3. Determine todos os inteiros n 1 para os quais 2n 1 divisvel por 7. 4. Ache o resto da diviso por 7 da soma 1! + 2! + + 50!. 5. Seja p 1 um inteiro primo. Mostre que
2p p

2 mod p.
7

6. Calcule o algarismo das unidades de 77 e de 77 .


5.5 Congruncias e divisibilidade

Proposio 4 Sejam a, b, c, m, n Z, com m > 1 e n > 1.


(a) Se a b mod m e se n|m, ento a b mod n; (b) a b mod m e a b mod n a b mod [m, n]; (c) Se a c b c mod m e (c, m) = 1, ento a b mod m; (d) Se d = (c, m), ento a c b c mod m a b mod
m d.

A vericao algo fastidiosa, mas no complicada...! (a) Como m|(a b), ento, se n|m, teremos n|(a b), donde a b mod n; (b) Se a b mod m e a b mod n, ento m|(a b) e n|(a b), logo, pela denio do mmc, segue [m, n]|(a b), donde a b mod [m, n]. Reciprocamente, se a b mod [m, n], ento, de (a), vemos que a b mod m e a b mod n, pois m|[m, n] e n|[m, n]. (c) Se m|(ac bc) = (a b)c e (c, m) = 1, ento m|(a b).

56

Introduo Teoria dos Nmeros  [Antonio Carlos & Ana Paula Marques]

(d) Pondo c = c1 d e m = m1 d, temos (c1 , m1 ) = 1. Se a c b c mod m, ento ac bc = tm para algum t Z, donde ac1 bc1 = tm1 ; como (c1 , m1 ) = 1, vem a b mod m1 . Reciprocamente, se a b mod m1 e a b = tm1 , ento ac bc = tcm1 = tc1 dm1 = tc1 m, logo a c b c mod m. Por exemplo, compare os cancelamentos: 42 7 mod 5, (5, 7) = 1, logo 42/7 7/7 mod 5, ou 6 1 mod 5;

50 20 mod 15, (10, 15) = 5, logo 50/10 20/10 mod 15/5, ou 5 2 mod 3.

Proposio 5  Critrios de divisibilidade


Seja a = an an1 . . . a2 a1 a0 um inteiro positivo cuja representao decimal a = an 10n + + a2 102 + a1 10 + a0 , onde 0 ak 9 e an = 0. Temos (a) a divisvel por 2, 5, 10 a a0 0 (mod 2, 5, 10); (b) a divisvel por 3, 9 a an + an1 + + a1 + a0 0 (mod 3, 9); (c) a divisvel por 4 a (a1 , a0 ) = 10a1 + a0 0 (mod 4) (d) a divisvel por 6 a 4(an + an1 + + a1 ) + a0 0 (mod 6); (e) a divisvel por 8 a (a2 , a1 , a0 ) = 100a2 + 10a1 + a0 0 (mod 8) (f) a divisvel por 11 a1 + a3 + a0 + a2 + (mod 11). Os critrios decorrem dos valores das potncias de 10 em relao aos divisores considerados. Assim: (a) Para cada n 1, temos 10n 0 mod 2, 5, 10, donde a a0 (mod 2, 5, 10); (b) Como 10 1 (mod 3, 9), segue 10n 1 (mod 3, 9), para cada n 1, donde a an + a2 + a1 + a0 (mod 3, 9); (c) Aqui, vale 10n 0 mod 4 se n 2, donde a (a1 , a0 ) mod 4; (d) Nesse caso, se n 1, temos 10n 4 mod 6; assim, a 4(an + + a2 + a1 ) + a0 (mod 6). (e) Basta ver que 1000 = 103 0 mod 8, o que anula ( mod8) as demais potncias de 10. (f) Agora, as potncias de 10 valem, alternadamente, 1 e 1: 10 1, 102 1, 103 1 mod 11,... Segue o critrio: um inteiro divisvel por 11 se, e s se, a diferena entre a soma dos algarismos de ordem mpar e a soma dos algarismos de ordem par for divisvel por 11.
Pratique um pouco!

Determine os algarismos x, y e z em cada caso para que os nmeros abaixo, representados na base 10, tenham a propriedade mencionada: (a) 2x7y divisvel por 44; (c) 45xy divisvel por 4 e por 9; (b) 28x75y divisvel por 33; (d) 13xy 45z divisvel por 8, por 9 e por 11.

Aplicao  Sistemas de resduos


Um sistema completo de resduos mdulo m qualquer conjunto de m nmeros naturais, cujos restos pela diviso por m so os nmeros 0, 1, . . . , m 1, sem repeties e numa ordem qualquer. claro que, se a1 , a2 , . . . , am so m nmeros naturais, dois a dois no congruentes mdulo m, ento eles formam um sistema completo de resduos mdulo m. Mais presisamente:

Proposio 6 . Se {a1 , a2 , . . . , am } um sistema completo de resduos mdulo m, seja k tal que


(k, m) = 1. Ento, para cada inteiro a, o conjunto {a + ka1 , . . . , a + kam } tambm um sistema completo de resduos mdulo m.
De fato, basta ver que a + ka1 , . . . , a + kam so, dois a dois, no congruentes (mod m).

Aritmtica Modular

57

5.6

O Pequeno Teorema de Fermat

Fixado um nmero primo p 2, ento ap a (mod p), para todo inteiro a Z. Usaremos o teorema da induo nita, supondo, inicialmente, que a 1. O resultado vale para a = 1, pois 1p = 1; se a 1 e ap a (mod p), mostremos que (a + 1)p a + 1 (mod p). Aqui, o argumento crucial resulta do binmio de Newton: se a, b so inteiros, ento (a + b)p ap + bp (mod p) (*) De fato, a expresso usual do binmio de Newton mostra que p1 p pk k p p a b + bp (a + b) = a + k
k=1

Ora, j vimos que p divide todos os binomiais o argumento (*) acima enunciado.

p , isto , k

p1

k=1

p pk k a b 0 mod p, o que garante k

Enm, temos (a + 1)p ap + 1p (mod p), donde, usando a hiptese de induo ap a (mod p), obtemos bem (a + 1)p a + 1 (mod p), valendo o resultado para todo inteiro a 1. Para completar o teorema, ainda falta examinar o caso a < 0, j que a = 0 no oferece diculdade. Pondo = a > 0 e supondo p 3 mpar, vale, como vimos, a congruncia p , ou (a)p (a), isto , ap a ou ap a mod p. Para encerrar mesmo, ainda falta ver o caso do primo p = 2, sempre com a < 0; agora, a equao 2 ca a2 a (mod 2), que verdadeita, pois a2 + a = a(a + 1) mltiplo de 2.

Corolrio Seja p 2 primo. Se p no divide o inteiro a, ento ap1 1 (mod p). De fato, se p |a, ento (p, a) = 1 e o resultado segue da proposio 4.(c).
Tanto o Pequeno Teorema, como seu corolrio, s vezes, so formulados em termos de divisibilidade:

Dado um primo p 2 e se a qualquer inteiro, ento p|ap a; Se p 2 primo e (p, a) = 1, com a inteiro, ento p|ap1 1.

Exemplo 5.7 1. Veriquemos que, dado um inteiro n > 0, ento n9 e n tm o mesmo algarismo da unidade.
Essa armao equivale a 10|n9 n; ou, como 10 = 2 5 e (2, 5) = 1, basta ver que 2|n9 n e 5|n9 n. Ora, n9 e n tm mesma paridade, logo n9 n par, donde mltiplo de 2. Por outro lado, temos n9 n = n(n8 1) = n(n4 1)(n4 + 1) = (n5 n)(n4 + 1) Pelo Teorema de Fermat, temos 5|(n5 n), portanto 5|n9 n. Assim, 10|n9 n.

2. Use o Pequeno Teorema de Fermat para calcular o resto da diviso


(a) de 11p1 por p se p primo; (b) de 2100 por 11; (c) de 23728 por 13 (a) Se p = 11 primo, ento (p, 11) = 1, logo, por Fermat, 11p1 1 mod p. Se p = 11, ento 1110 0 mod 11; (b) Pelo Teorema, 210 1 (mod 11), logo, 2100 = (210 )10 1 (mod 11); (c) Usando o Teorema, 23712 1 mod 13, donde 23724 1 mod 13, logo 23728 2374 mod 13. Como 237 3, vem 2374 34 = 81 3 mod 13.

58

Introduo Teoria dos Nmeros  [Antonio Carlos & Ana Paula Marques]

3. Com o auxlio do Teorema de Fermat, vericar que inteiro o nmero

3n5 + 5n3 + 7n . 15 3n5 + 5 n3 + 7 n n5 n n3 n 8n 7n n5 n n3 n Temos = + + + = + + n. Ora, pelo Teorema, 15 5 3 15 15 5 3 5|n5 n e 3|n3 n; assim, a frao inicial bem um inteiro.

4. Seja Fn = 22 + 1, n 1, o ensimo nmero de Fermat.


n

(a) Usando induo sobre n 1, mostrar que 3|Fn 2, n 1; (b) Concluir que Fn no um quadrado pefeito. (a) Se n = 1, ento F1 = 5 e F1 2 = 3, logo 3|F1 2. Supondo n 1 e o resultado vlido para Fn , veriquemos a validade para Fn+1 . Temos n+1 n 2 2 2F + 2, isto , F Fn+1 = 22 + 1 = 22 + 1 = (Fn 1)2 + 1 = Fn n n+1 2 = Fn (Fn 2) Da hiptese de induo, 3|Fn 2,logo, da ltima igualdade acima, vem 3|Fn+1 2. (b) Se existe a Z tal que Fn = a2 , ento seria a2 2 0 (mod 3), ou a2 2 (mod 3), o que impossvel, j que h somente trs tipos de restos (0, 1, ou 2 mod3), cujos quadrados so 0 ou 1.

5. Se p e q so primos distintos, ento pq1 + q p1 1 (mod pq ).


Basta aplicar o Teorema para p, depois para q : pq1 + q p1 1 (mod p), pq1 + q p1 1 (mod q ) Como (p, q ) = 1, segue o resultado enunciado.

Atividade-proposta 5.8
1. Partindo de 1001 = 7 11 13, se a = an 10n + + a2 102 + a1 10 + a0 (escrito na base 10), ento a divisvel por 7,11,13 a5 a4 a3 + a11 a10 a9 + a2 a1 a0 + a8 a7 a6 + (mod 7, 11, 13) 2. Use o Pequeno Teorema de Fermat para calcular o resto da diviso (a) de 14256 por 17; (b) de 521 por 127; (c) de 17 + 27 + + 1007 por 7. 3. Para cada inteiro n 1, mostre que: (a) 198n 1 (mod 17); (b) 270 + 370 0 (mod 13); (c) 1016n divisvel por 70 4. Com o auxlio do Pequeno Teorema de Fermat, vericar que inteiro o nmero 5. Seja Fn = 22 + 1, n 2, o ensimo nmero de Fermat. (a) Usando induo sobre n 2, mostre que 22 6 (mod 10); (b) Concluir que o algarismo da unidade de Fn igual a 7, para todo n 2. 6. Encontre os possveis valores do inteiro n 1, para os quais o nmero tn = 2n + 1 divisvel por 2; 3; 5 e 15, respectivamente. 7. Seja o primo positivo p = 2, 3. Mostre que (a) p 1 (mod 6) ou p 5 (mod 6); (b) p2 1 (mod 24).
n n

12n13 23n + . 35 35

Aritmtica Modular

59

5.9

Anel

Zm

dos inteiros mod

Vimos que a congruncia (mod m) uma relao de equivalncia em Z. As respectivas classes de equivalncia so denominadas classes residuais mdulo m; assim, a classe residual do elemento a Z o conjunto a = {x Z ; x a mod m}. As propriedades abaixo valem para quaisquer classes de equivalncia.

Proposio 7. Seja um inteiro m 1.


(a) Toda classe residual mdulo m no vazia; (b) a = b se e somente a b (mod m); (c) Se a b = , ento a = b; (d)
aZ

a = Z.

A vericao no oferece diculdade. (a) Como a a, sempre vale a a; (b) se a = b, como a a, segue que a b, logo a b. Reciprocamente, suponhamos a b; temos: x a x b, logo a = b; a Z; (c) se x a b, ento x a e x b, logo a b, donde, por (b), a = b; em (d), claro que
aZ

reciprocamente, se x Z, como x x, ento x


aZ

a e portanto Z
aZ

a.

Observao Um inteiro qualquer b tal que b = a um representante da classe a. Proposio 8. Para cada a Z, existe um e somente um r Z, com 0 r < m tal que a = r.
Pela diviso euclidiana, existe um nico resto 0 r < m tal que a = mq + r ou a r mod m.

Corolrio Existem exatamente m classes residuais mdulo m distintas, a saber 0, 1, . . . , m 1.


Na realidade (veja a denio que precede a Proposio 6), {a1 , a2 , . . . , am } um sistema completo de resduos mdulo m se, e somente se, a1 , a2 , . . . , am so as m classes residuais mdulo m. Exempliquemos os aspectos mencionados.

Se m = 2, ento 0 = {x Z ; x 0 mod 2} = {x Z ; x par}; 1 = {x Z ; x 1 mod 2} = {x Z ; x mpar}. Temos tambm que a = 0, se a par e a = 1, se a mpar. Assim, qualquer inteiro par representante da classe 0 e qualquer inteiro mpar representante da classe 1. Se m = 3, ento as classes residuais so {0, 1, 2}, onde 0 = {3q ; q Z}, e a = 0 se a mltiplo de 3; 1 = {3q + 1 ; q Z}, e a = 1, se a tem resto 1 na diviso por 3; 2 = {3q + 2 ; q Z}, e a = 2, se a tem resto 2 na diviso por 3. Qualquer mltiplo de 3 representante de 0. Temos que . . . , 8, 5, 2, 1, 4, 7, . . . so representantes da classe 1, enquanto . . . , 4, 1, 2, 5, 8, . . . so representantes de 2.
A estrutura de anel

O conjunto Zm = {0, 1, , m 1} de todas as classes residuais mdulo m ser munido de uma rica estrutura aritmtica, que denominaremos anel dos inteiros mdulo m. Em Zm denimos as operaes:

Adio: a + b = a + b Multiplicao: a b = a b
Tais operaes foram denidas via representantes das classes a e b; para ver que as operaes no dependem dos representantes escolhidos, basta reler a Proposio 3(d)(e).

60

Introduo Teoria dos Nmeros  [Antonio Carlos & Ana Paula Marques]

Propriedades da Adio . Para todos a, b, c Zm , temos


A1 (Associatividade) (a + b) + c = a + (b + c); A2 (Comutatividade) a + b = b + a; A3 (Existncia de zero) a + 0 = a; A4 (Existncia de simtrico) a + a = 0.

Propriedades da Multiplicao . Para todos a, b, c Zm , temos


M1 (Associatividade) (a b) c = a (b c); M2 (Comutatividade) a b = b a; M3 (Existnia de unidade) a 1 = a;

Propriedade Distributiva . Para todos a, b, c Zm , temos


a (b + c) = a b + a c;
Observando essas propriedades, percebemos uma certa semelhana com aquelas dos nmeros inteiros. Mas ateno! Ocorrem grandes diferenas, como passaremos a examinar, principalmente com a multiplicao. Inicialmente, olhemos com ateno as tabelas da adio e da multiplicao abaixo. a) Z2 = {0, 1}

+ 0 1 0 1
b) Z3 = {0, 1, 2}

0 1

0 1 1 0

0 0 0 1 0 1 0 1 2

+ 0 1 2 0 1 2
c) Z4 = {0, 1, 2, 3}

0 1 2 1 2 0 2 0 1

0 0 0 0 1 0 1 2 2 0 2 1 0 1 2 3 0 0 0 0 0 1 2 3 0 2 0 2 0 3 2 1

+ 0 1 2 3 0 1 2 3
d) Z5 = {0, 1, 2, 3, 4}

0 1 2 3

1 2 3 0

2 3 0 1

3 0 1 2

0 1 2 3

+ 0 1 2 3 4 0 1 2 3 4 0 1 2 3 4 1 2 3 4 0 2 3 4 0 1 3 4 0 1 2 4 0 1 2 3

0 1 2 3 4 0 0 0 0 0 0 1 2 3 4 0 2 4 1 3 0 3 1 4 2 0 4 3 2 1

0 1 2 3 4

A multiplicao de Z2 , Z3 , e Z5 , bem estruturada, semelhana do produto de Q ou de R, em que todo elemento x = 0 possui um inverso multiplicativo y , tal que xy = 1; por exemplo, em Z5 , 2 3 = 1, e tambm, 4 4 = 1. J o Z4 meio esquisito: apenas 1 e 3 so inversveis; 2 = 0, mas 2 2 = 0, isto , no vale a propriedade de integridade; 2 1 = 2 = 2 3, mas 1 = 3, no valendo, assim, o cancelamento multiplicativo restrito. Na realidade, veremos que a boa aritmtica de um Zm ocorre quando m primo!!

Aritmtica Modular

61

5.10

Elementos inversveis de

Zm

Um elemento a de Zm inversvel se existe b Zm tal que a b = 1. Observemos que, se existe um tal b, ento esse elemento unicamente determinado por a; de fato se c Zm tambm verica a c = 1, ento c = c 1 = c (a b) = (c a) b = 1 b = b Assim se a inversvel, falaremos do seu inverso, digamos b. A unicidade do inverso tambm nos d b inversvel, com inverso a.

Notaremos U (m) o conjunto dos elementos inversveis de Zm .


Por exemplo, U (2) = {1}, U (3) = {1, 2}, U (4) = {1, 3}, U (5) = {1, 2, 3, 4}, U (8) = {1, 3, 5, 7}.

O conjunto U (m) multiplicativamente estvel: se a, b Zm , ento a b Zm .


Realmente, se a e b so inversveis, com a = 1 e b = 1, ento o inverso de a b :

(a b)( ) = (a )(b ) = 1 1 = 1
logo a b inversvel.

A multiplicao de Zm , desse modo, induz uma lei de composio interna. : U (m) U (m) U (m), (x, y ) x y
O produto de comutativo, associativo, possui unidade multiplicativa 1 e cada elemento posssui um inverso multiplicativo. O prximo resultado caracteriza os elementos inversveis de Zm .

Proposio 9 Um elemento a Zm inversvel se e somente (a, m) = 1.


De fato, se a inversvel, ento existe b Zm tal que a b = ab = 1, logo ab 1 (mod m), donde existe um inteiro t tal que ab 1 = mt, ou ab mt = 1,logo (a, m) = 1. Reciprocamente, se (a, m) = 1, ento existe uma combinao linear de Bzout a + m t = 1, donde a = 1,o que mostra ser a inversvel. Observemos que o algoritmo de Euclides do MDC decide a inversibilidade e, no bom caso (a, m) = 1, explicita o inverso multiplicativo!!

Pratique um pouco! Para decidir se 3640 inversvel em Z7297 , usamos o algoritmo de Euclides,
do qual obtemos o mdc(7297, 3640) = 1 e a combinao linear 1713 7297 3434 3640 = 1; logo, 3640 inversvel no anel considerado e seu inverso vale 3434 3863 (mod 7297).

A Proposio 9 nos informa que tipo de Zm possui estrutura de corpo.

Proposio 10 Zm um corpo se e somente se m primo.


De fato, se m primo, ento, para cada 1 j m 1, temos (j, m) = 1, donde todos os elementos no nulos, pela Proposio 9, 1, 2, . . . , m 1 so inversveis. Reciprocamente, se Zm um corpo e m no primo, ento m = m1 m2 , com 1 < m1 , m2 < m, o que acarreta 0 = m = m1 m2 com m1 = 0 e m2 = 0, contradio.

O argumento acima explica parte das diculdades aritmticas de um Zm , com m no primo: a existncia dos chamados divisores de zero, elementos para os quais no vale a propriedade de integridade, isto , um elemento x no nulo, para o qual existe um elemento y , tambm no nulo, mas com x y = 0!!

Pratique um pouco!
Observe em Z6 : U (6) = {1, 5}; os demais elementos 2, 3, 4 so divisores de zero: 2 3 = 4 3 = 0.

62

Introduo Teoria dos Nmeros  [Antonio Carlos & Ana Paula Marques]

Exemplo 5.11

1. Vericar que um elemento a Zm um divisor de zero se e somente se (a, m) = d > 1.


Realmente, se vale d > 1, seja a = a1 d e m = m1 d; temos 1 m1 < m, logo m1 = 0 e, por outro lado, am1 = (a1 d)m1 = a1 (dm1 ) = a1 m, isto , a m1 = a1 m = 0, portanto a um divisor de zero. Reciprocamente, suponhamos que a seja um divisor de zero: existe b = 0 tal que ab = 0, donde m|ab. Por absurdo, se fosse (a, m) = 1, ento a e m seriam primos relativos e de m|ab resultaria m|b, logo b = 0, contra a hiptese.

2. Encontrar todas as solues das equaes dadas no anel Zm dado.


(a) 3x + 2 = 6x + 7, m = 8; Temos 6x 3x = 2 7; 3x = 5 = 3. Ora, 3 inversvel em Z8 , com inverso igual a 3. Multiplicando ambos membros da ltima congruncia por 3, segue x = 1. Observao. Tambm podemos usar a Proposio 4(c). (b) x2 = 1, m = 7, m = 8; Como x2 1 = (x 1) (x + 1) e m = 7 primo, ento, como Z7 um corpo, logo um domnio de integridade, vem x 1 = 0 ou x + 1 = 0, isto , x = 1 ou x = 1 = 6. Em Z8 , a mesma igualdade x2 1 = (x 1)(x + 1) mostra que pode ser x = 1 e x = 1 = 7. Mas, ser que no h outras solues? Observando os quadrados mod 8, vemos que podem ocorrer outros valores, como x = 3 e x = 5. x 0 1 2 3 4 5 6 7

x2 0

Observao. Assim, em Z8 , a equao do segundo grau x2 = 1 possui quatro raizes!!

3. Considere o grupo multiplicativo U (9) = {1, 2, 4, 5, 7, 8}. Observemos que as potncias sucessivas de 2 fornecem todos os elementos de U (9); diremos, ento, que U (9) gerado por 2:
21 = 2; 22 = 4; 23 = 8; 24 = 16 7 (mod 9); 25 2 7 14 5, logo 26 10 1 (mod 9). Pratique um pouco! Vericar que, alm de 2, apenas 5 tambm gerador de U (9).
Atividade-proposta 5.12

1. Se p 2 primo, ento no h divisor de zero em Zp . Observao. Na realidade, o resultado vale num corpo qualquer. 2. Faa a lista dos elementos de U (7) e U (8), bem como as respectivas tabelas multiplicativas. Em Z7 e Z8 h divisores de zero? 3.Considere o anel Z24 . (a) Determine os elementos inversveis de Z24 e os divisores de zero desse anel; (b) Para cada elemento inversvel, ache o respectivo inverso e construa a tabela do grupo multiplicativo U (24); (c) Para cada a = 0 que divisor de zero, ache b = 0 tal que a b = 0. 4. Ache todas as solues das equaes dadas no anel Zm dado. (a) 4x 7 + 6x + 2 = 3x + 5, m = 12; (b) x3 = x, m = 8; (c) 4x2 + 5x + 7 = 0, m = 11, m = 12.

Aritmtica Modular

63

5. O resto da diviso de um inteiro a > 0 por 8 igual a 5, e o resto da diviso de a por 11 vale 4. Ache o resto da diviso de a por 88. 6. No grupo multiplicativo Z 7 = U (7) = Z7 {0}, identique cada elemento cujas potncias descrevem todo o grupo. [Cf. o exemplo 5.11(3) acima].

5.13 O indicador de Euler

A Proposio 9 nos d a seguinte descrio alternativa: U (m) = {a Zm ; (a, m) = 1}. Pois bem. O nmero de elementos de U (m) dene uma funo aritmtica fundamental, a saber, o indicador de Euler (). Por denio, temos : N N, (1) = 1; se m 2, (m) = # U (m) = # {1 a < m ; (a, m) = 1}
Em particular, para cada m 2, temos (m) m 1. Por exemplo, (2) = 1, (3) = 2, (4) = 2, (5) = 4, (6) = 2, (7) = 6, (8) = 4. Sistema reduzido de resduos mdulo m Partindo de um sistema completo qualquer de resduos a1 , . . . , am mdulo m, se eliminarmos os elementos aj que no so primos com m, obtemos um sistema reduzido de resduos mdulo m, isto , um conjunto de (m) de inteiros r1 , r2 , . . . , r(m) tais que (a) (rj , m) = 1, para todo j tal que 1 j (m) (b) ri rj se i = j ; (c) Para cada inteiro n tal que (n, m) = 1, existe j tal que n rj mod m.

Proposio 11 Seja a um inteiro positivo tal que (a, m) = 1. Se r1 , r2 , . . . , r(m) um sistema reduzido de resduos mdulo m, ento ar1 , ar2 , . . . , ar(m) tambm um sistema reduzido de resduos mdulo m.
De fato, como (a, m) = 1 e (rj , m) = 1, ento (arj , m) = 1; alm disso, de ari arj (mod m), segue ri rj , logo i = j .

Teorema (Euler)
Seja m 2; para cada inteiro positivo a tal que (a, m) = 1, vale a(m) 1 (mod m) . Vimos acima que ar1 , ar2 , . . . , ar(m) um sistema reduzido de resduos mdulo m, se (a, m) = 1, e r1 , r2 , . . . , r(m) for um sistema reduzido de resduos mdulo m. Como cada arj congruente a exatamente um dos rj , o produto dos arj deve ser congruente ao produto dos rj : ar1 ar2 ar(m) r1 r2 r(m) (mod m)

a(m) r1 r2 r(m) r1 r2 r(m) (mod m) Enm, como (r1 r2 r(m) , m) = 1, o produto dos rj pode ser cancelado, donde a(m) 1 (mod m).
Observemos a generalizao do Pequeno Teorema de Fermat: dado um primo p 2, se (a, p) = 1, isto , se p |a, ento, como (p) = p 1, reencontramos ap1 1 (mod p). Para bem usufruir do Teorema de Euler, vamos logo calcular, explicitamente, cada (m), para m 2. A idia usar a decomposio primria de m. Para tanto, vamos iniciar com o seguinte fato.

Dado um nmero primo p 2, para cada 1, temos

(p ) = p p1 .

De fato, temos que excluir, de 1 at p , os nmeros que no so primos com p , ou seja, todos os mltiplos de p, a saber, p, 2p, . . . , p1 p, cujo nmero p1 . Donde: (p ) = p p1 .

64

Introduo Teoria dos Nmeros  [Antonio Carlos & Ana Paula Marques]

Em seguida, veriquemos que multiplicativa: se (a, b) = 1, ento (a b) = (a) (b).


A tabela dos nmeros naturais de 1 at ab pode ser assim organizada:

1 b+1 . . .

2 b+2 . . .

k b+k . . .

b 2b . . . ab

(a 1)b + 1 (a 1)b + 2

(a 1)b + k

Como (t, ab) = 1 (t, a) = 1 e (t, b) = 1, para calcular (a b), temos que achar os inteiros da tabela que so simultaneamente primos com a e b. Se o primeiro elemento de uma coluna no for primo com b, ento todos os elementos da coluna no so primos com b. Portanto, os elementos primos com b esto necessariamente nas colunas restantes, que so em nmero (b), cujos elementos so primos com b. Agora, h (a) elementos primos com a em cada uma dessas colunas: basta observar que, como (a, b) = 1, a sequncia [veja a k-sima coluna acima] b, b + k, . . . , (a 1)b + k forma um sistema completo de resduos mdulo a. Logo, o nmero de elementos que so simultaneamente primos com a e b (a) (b).

Teorema  Clculo do indicador de Euler n 1 Dado o inteiro m 1 e sua decomposio primria m = p 1 pn , temos
n 1 1 (m) = p 1 pn

1 p1

1 pn

=m 1

1 p1

1 pn

=m
p|m

1 p

n 1 De fato, temos (m) = (p 1 ) (pn ) donde o resultado, pois, para cada 1 j n, vale

(pj j ) = pj j pj j
A expresso acima tambm pode ser escrita

= pj j

1 pj

1 1 n 1 n 1 (p 1) (p 1). (m) = (p p 1 n n 1 pn ) = p1

Exemplo 5.14

1. Alguns exemplos numricos de indicadores de Euler.


(a) Se p 2 primo, ento (p) = p 1, pois os p 1 nmeros {1, 2, . . . , p 1} so menores do que p e primos com p; (b) (8) = (23 ) = 23 22 = 4; (c) (10) = (2 5) = (2) (5) = 1 4 = 4; (d) (7865) = (5 112 13) = 4 (112 11) 12 = 4 110 12 = 5280;
1 1 )(1 1 (e) (1350) = (2 33 52 ) = 1350(1 2 3 )(1 5 ) = 360.

2. Veriquemos que um inteiro n 2 primo se, e somente se, (n) = n 1.


O caso n primo direto, como vimos em 1(a), acima. Reciprocamente, se fosse n composto, ento existiria algum divisor no trivial de n e no primo com n, o que acarretaria (n) n 2, contra a hiptese.

3. Se o inteiro m 3, ento (m) um inteiro par.


De fato, se m uma potncia de 2, m = 2k , com k 2, ento (m) = (2k ) = 2k 2k1 = 2k1 par. Se m no uma potncia de 2, ento m divisvel por um primo mpar p, m = pk q e (pk , q ) = 1.

Aritmtica Modular

65

Logo, vemos que (m) = (pk q ) = (pk )(q ) = (pk pk1 )(q ) = pk1 (p 1)(q ), o que mostra, ainda nesse caso, (m) par, pois 2|(p 1).

4. Sejam os inteiros a 2 e m > 0. Se (a, m) = (a 1, m) = 1, ento


1 + a + a2 + + a(m)1 0 (mod m). Como (a, m) = 1, podemos usar o Teorema de Euler: a(m) 1 (mod m); ora, por outro lado, temos: a(m) 1 = (a 1)(a(m)1 + a(m)2 + + a + 1); enm, a condio (a 1, m) = 1 permite cancelar o fator (a 1) mod m.

5. Seja p 3 um primo mpar, tal que 2p +1 tambm primo. Se m = 4p, ento (m +2) = (m)+2.
Como p mpar, temos (4p) = (4)(p) = 2(p 1); tambm 2p + 1 um primo mpar, logo (4p + 2) = (2(2p + 1)) = (2p + 1) = 2p = 2 + (4p).

Observao Um primo p para o qual 2p + 1 tambm primo, conhecido como um primo de Sophie
Germain;

por exemplo, 2,3,5,11,23,29,41.

6. Para todo inteiro positivo m, vale (m2 ) = m(m).


De fato, (m2 ) = m2
p|m2

1 p

= m2
p |m

1 p

= m (m).

Por exemplo, (100) = (102 ) = 10(10) = 10 4 = 40.

7. Para cada inteiro m 2, a soma dos inteiros positivos menores que m e que so primos com m
igual a

1 m (m). 2 Sejam r1 , r2 , . . . r(m) os inteiros positivos menores que m e primos com m; como a condio (r, m) = 1 equivale a (m r, m) = 1, os inteiros considerados tambm podem ser expressos pelas diferenas m r1 , m r2 , . . . , m r(m) , isto :
(m) (m) (m) (m)

rj =
j =1 j =1

(m rj ) = m(m)
j =1

rj , donde o resultado enunciado


j =1

1 rj = m (m). 2

Aplicao numrica. Veriquemos a relao anterior para m = 15. Temos (15) = (3 5) = (3 1)(5 1) = 8, sendo U (15) = {1, 2, 4, 7, 8, 11, 13, 14} um conjunto de 1 1 resduos reduzidos mod15. Ento: 1 + 2 + 4 + 7 + 8 + 11 + 13 + 14 = 60 = 15 8 = 15 (15). 2 2
Atividade-proposta 5.15

1. Calcule (m), dados m = 17; m = 51; m = 100. 2. Encontre os inteiros m 1 para os quais (m) = 1; (m) = 2; (m) = m. 3. Exemplique: dois inteiros a e b para os quais (ab) = (a) (b). 4. Use o Teorema de Euler para achar o resto da diviso de 3100 por 34. Sugesto. Use o valor (34) = (2 17) = 16. 5. Dado um nmero perfeito par m, ento existe um inteiro k tal que (m) = 2k1 (2k1 1). Sugesto. Pelo Teorema de Euclides-Euler do 4.5, m da forma m = 2k1 (2k 1), onde 2k 1 um nmero primo (de Mersenne); calcule, ento (m). 6. Supondo que (m, n) = 1, ento m(n) + n(m) 0 (mod mn). 7. Todas as solues da equao

(m) 2 = so da forma m = 3k , k = 1, 2, . . .. m 3

66

Introduo Teoria dos Nmeros  [Antonio Carlos & Ana Paula Marques]

5.16 Teorema de Wilson

No captulo anterior (veja a Atividade 4.9, Exerccio 5), solicitamos a validao do seguinte resultado: Se p 2 um nmero primo, ento (p 1)! 1 (mod p). Na realidade, o resultado acima e sua recproca so conhecidos por Teorema de Wilson, mas que, na realidade, foram provados, anteriormente, por Lagrange. S nos falta, ento vericar que, se p primo, vale (p 1)! 1 (mod p). Ora, no corpo Zp , a equao x2 = 1 possui, exatamente, duas solues x = 1, o que signica que, dentre os elementos no nulos {1, 2, . . . , p 1}, apenas 1 e 1 p 1 so seus prprios inversos; os demais p 3 elementos se agrupam aos pares, cada um com seu inverso. Desse modo, no produto 1 2 3 (p 1) = (p 1)! s resta o produto 1 (p 1). Assim (p 1)! 1 p 1 (mod p). Aplicao numrica. O procedimento acima, exemplicado no corpo Z17 , fornece os 7 agrupamentos: (2, 9), (3, 6), (4, 13), (5, 7), (10, 12), (11, 14), (8, 15)

Exemplo
Se p um nmero primo mpar, ento 2p1 + (p 1)! 0 (mod p). De fato, sendo p mpar, temos (2, p) = 1, logo, pelo Pequeno Teorema de Fermat, 2p1 1 mod p. Por outro lado, do Teorema de Wilson, sabemos que (p1)! 1 mod p. Logo, temos bem 2p1 +(p1)! 0 (mod p).

Pratique um pouco! Use o Teorema de Wilson para mostrar que um inteiro m > 2 primo se, e
somente se, m divide o nmero 2(m 3)! + 1.
5.17 Resoluo de Congruncias Lineares

1. Introduo
Consideremos o seguinte problema: dados dois inteiros a e b e o inteiro m 2, determinar todos os nmeros inteiros x tais que ax b (mod m). Esse problema conhecido pelo nome de congruncia linear mdulo m. Um inteiro x0 soluo da congruncia ax b (mod m) (*) se ax0 b (mod m); o conjunto de todos os nmeros inteiros que satisfazem essa condio o conjunto-soluo S da congruncia (*). Observemos que, se x0 soluo e x1 x0 , ento x1 tambm soluo de (*), isto , todos os inteiros da classe x0 de restos determinada por uma soluo so solues da congruncia (*). Portanto, a descrio completa do conjunto-soluo S envolve classes de solues e, dentre elas, as que so incongruentes.

2. Clculo das solues


Retomando nossa congruncia (*), se existe x0 vericando ax0 b mod m, ento m|(ax0 b), isto , existe t Z tal que ax0 b = mt, ou ax0 mt = b, logo, a equao diofantina ax + (m)y = b admite o par ordenado (x0 , t) como soluo. Reciprocamente, claro que cada soluo da equao diofantina considerada tambm verica a congruncia (*). Tendo em conta nossa experincia com as equaes diofantinas do 3.25, podemos enunciar.

Teorema
A congruncia linear ax b (mod m) possui soluo se, e somente, o mdc d = (a, m) divide b. (a) Se x0 uma soluo, ento todas as solues x = x0 + m1 t, t Z, formam d classes mutuamente incongruentes mdulo m, assim representadas, onde m = m1 d: x0 , x0 + m1 , x0 + 2m1 ,. . . , x0 + (d 1)m1 .

Aritmtica Modular (b) Se (a, m) = 1, ento a congruncia ax b (mod m) possui um nica soluo mdulo m.
Em qualquer caso, uma soluo particular x0 dada por uma combinao linear de Bzout. Vamos logo vericar que os nmeros x0 + jm1 so solues:

67

a(x0 + jm1 ) = ax0 + ajm1 = ax0 + j (a/d)m ax0 b (mod m) Alm disso, esses nmeros so dois a dois incongruentes mdulo m, pois, para i, j < d, vem m x0 + im1 x0 + jm1 (mod m), donde im1 jm1 (mod m), ou como = d, resulta i j (m1 , m) (mod d), donde i = j .
Enm, veriquemos que toda soluo x da congruncia ax b (mod m) equivalente, modulo m, a algum x0 + jm1 , j < d. Como ax ax0 (mod m), vemos que x x0 (mod m1 ), donde x x0 = km1 ; ora, pela diviso euclidiana, k = qd + j , com j < d, portanto

x = x0 + qdm1 + jm1 = x0 + qm + jm1 x0 + jm1 (mod m) Observao Se d = (a, m)|b, coloquemos a = a1 d, b = b1 d e m = m1 d. As equaes ax b (mod m)(*) e a1 x b1 (mod m1 )(**) so equivalentes, pois, se x0 verica ax0 b (mod m), ento (veja 5.5Prop.4) a1 x0 b1 (mod m1 ), isto , x0 verica (**). Por outro lado, se z0 verica a1 z0 b1 (mod m1 ), ento a1 z0 = b1 + tm1 , com t Z, logo a1 dz0 = b1 d + tm1 d ou az0 = b + tm, donde az0 b (mod m). Desse modo, as congruncias lineares (*) e (**) possuem o mesmo conjunto-soluo. Entretanto, as solues da congruncia original (*) so consideradas mdulo m, enquanto as da congruncia reduzida (**) so mdulo m1 .
Exemplos 5.18

Resolver as seguintes congruncias lineares. 1. 5x 6 (mod 12) Temos (5, 12) = 1, logo a equao dada possui uma nica soluo mdulo 12; tambm, 5 inversvel em Z12 , com 5 5 = 25 1 mod 12. Multiplicando por 5 a congruncia dada: 25x 30 (mod 12), ou x 6 (mod 12). 2. 8x 4 (mod 12) Aqui, (8, 12) = 4 e 4 divide o segundo membro, logo a equao dada possui d = 4 solues mdulo 12. Formando a equao reduzida 2x 1 (mod 3), como 2 inversvel (mod 3), com 2 2 1 mod 3, ento, multiplicando a equao por 2, vem x 2 (mod 3), nica soluo mdulo 3. Como x = 2 + 3t, t Z, seguem as quatro solues incongruentes mdulo 12: 2, 2 + 3, 2 + 6, 2 + 9. 3. 3x 25 (mod 9) Temos que (3, 9) = 3 no divide 25. Logo, a equao no tem soluo em Z. 4. 18x 30 (mod 42) O mdc d = (18, 42) = 6 divisor de 30, logo h d = 6 solues incongruentes mdulo 42. A equao reduzida 3x 5 (mod 7) nos d x 4 (mod 7), nica soluo mdulo 7. Tendo em conta que x = 4 + 7t, t Z, seguem as seis solues incongruentes mdulo 42: x = 4, 11, 18, 25, 32, 39.
Atividade-proposta 5.19

Resolver as congruncias lineares.

1. 35x 5 (mod 14) 2. 64x 16 (mod 84) 3. 105x 4 (mod 167) 4. 315x 12 (mod 501)

Vous aimerez peut-être aussi